Vous êtes sur la page 1sur 34

ARTICLE VII- EXECUTIVE DEPARTMENT

1. MARCOS VS. MANGLAPUS


(177 SCRA 668)

FACTS: In February 1986, Ferdinand Marcos was deposed from the presidency via the non-violent ―people power‖
revolution and forced into exile. Corazon C. Aquino was declared President of the Philippines under a revolutionary
government.
After three years, Mr. Marcos, in his deathbed, has signified his wish to return to the Philippines to die. But
President Aquino has stood firmly in the decision to bar the return of Mr. Marcos and his family - considering the dire
consequences to the nation of his return at a time when the stability of the government is threatened from various
directions and the economy is just beginning to rise and move forward.

ISSUE: Whether or not in the exercise of the powers granted by the Constitution, the President may prohibit the
Marcoses from returning to the Philippines.

HELD: Yes, the President has the Power under the Constitution to bar the Marcoses from returning to our country.
The Constitution says that the executive power shall be vested in the President. It also enumerates certain specific
powers. The enumeration, however, does not exhaust the totality of executive powers. Tradition recognizes that the
powers of the President are more than the sum of enumerated executive powers. The duty of the government ―to serve
and protect the people‖ as well as to see to the ―maintenance of peace and order, the protection of life, liberty, and
property, and the promotion of the general welfare‖ argue towards the existence of ―residual instated powers.‖

1a. MARCOS VS. MANGLAPUS


(178 SCRA 760)

FACTS: On October 1989, motion for reconsideration was filed by petitioners raising the following major arguments
among others:
1. The President has no power to bar a Filipino from his own country.
2. There is no basis for barring the return of the family of former President Marcos.

ISSUE: W/N the President has the power to bar the Marcoses from returning to the Philippines.

HELD: YES. The Supreme Court held that it cannot be denied that the President, upon whom executive power is vested,
has unstated residual powers which are implied from the grant of executive power and which are necessary for her to
comply with her duties under the Constitution.
The powers of the President are not limited to what are expressly enumerated in the article on the Executive
Department and in scattered provisions of the Constitution. This is so, notwithstanding the avowed intent of the members
of the Constitutional Commission of 1986 to limit the powers of the President as a reaction to the abuses under the regime
of Mr. Marcos, for the result was a limitation of specific powers of the President, particularly those relating to the
commander-in-chief clause, but not a diminution of the general grant of executive power.

2. SOLIVEN V MAKASIAR
167 SCRA 393 – Political Law – Constitutional Law – President’s Immunity From Suit – Must Be Invoked by the President

Luis Beltran is among the petitioners in this case. He, together with others, was charged with libel by the then
president Corzaon Aquino. Cory herself filed a complaint-affidavit against him and others. Makasiar averred that Cory
cannot file a complaint affidavit because this would defeat her immunity from suit. He grounded his contention on the
principle that a president cannot be sued. However, if a president would sue then the president would allow herself to be
placed under the court’s jurisdiction and conversely she would be consenting to be sued back. Also, considering the
functions of a president, the president may not be able to appear in court to be a witness for herself thus she may be
liable for contempt.

ISSUE: Whether or not such immunity can be invoked by Beltran, a person other than the president.
HELD: No. The rationale for the grant to the President of the privilege of immunity from suit is to assure the exercise of
Presidential duties and functions free from any hindrance or distraction, considering that being the Chief Executive of the
Government is a job that, aside from requiring all of the office-holder’s time, also demands undivided attention.
But this privilege of immunity from suit, pertains to the President by virtue of the office and may be invoked only
by the holder of the office; not by any other person in the President’s behalf. Thus, an accused like Beltran et al, in a
criminal case in which the President is the complainant cannot raise the presidential privilege as a defense to prevent the
case from proceeding against such accused.
Moreover, there is nothing in our laws that would prevent the President from waiving the privilege. Thus, if so
minded the President may shed the protection afforded by the privilege and submit to the court’s jurisdiction. The choice
of whether to exercise the privilege or to waive it is solely the President’s prerogative. It is a decision that cannot be
assumed and imposed by any other person.

3. REVIEW CENTER ASSOC. OF THE PHILS. V EDUARDO ERMITA

Facts: On 11 and 12 June 2006, the Professional Regulation Commission (PRC) conducted the Nursing Board
Examinations nationwide. In June 2006, licensure applicants wrote the PRC to report that handwritten copies of two sets
of examinations were circulated during the examination period... among the examinees reviewing at the R.A. Gapuz
Review Center and Inress Review Center.
George Cordero, Inress Review Center's President, was then the incumbent President of the Philippine Nurses
Association.
On 18 August 2006, the Court of Appeals restrained the PRC from proceeding with the oath-taking of the
successful examinees set on 22 August 2006.
Consequently, President Gloria Macapagal-Arroyo (President Arroyo) replaced all the members of the PRC's Board of
Nursing. President Arroyo also ordered the examinees to re-take the Nursing Board Examinations.
On 8 September 2006, President Arroyo issued EO 566 which authorized the CHED to supervise the
establishment and operation of all review centers and similar entities in the Philippines.
the Review Center Association of the Philippines (petitioner), an organization of independent review centers, asked the
CHED to "amend, if not withdraw" the IRR arguing, among other things, that giving permits to... operate a review center to
Higher Education Institutions (HEIs) or consortia of HEIs and professional organizations will effectively abolish
independent review centers.
EO 566- authorized ched to supervise the establishment and operation of all review centers
In a letter dated 3 January 2007,[6] Chairman Puno wrote petitioner, through its President Jose Antonio Fudolig
(Fudolig), that to suspend the implementation of the IRR would be inconsistent with the mandate of EO 566.
On 7 May 2007, the CHED approved the RIRR. On 22 August 2007, petitioner filed before the CHED a Petition to
Clarify/Amend Revised Implementing Rules and Regulations[8] praying for a ruling:
Amending the RIRR by excluding independent review centers from the coverage of the CHED;
Clarifying the meaning of the requirement for existing review centers to tie-up or be integrated with HEIs, consortium or
HEIs and PRC-recognized professional associations with recognized programs, or in the alternative, to convert into
schools; and
Revising the rules to make it conform with Republic Act No. 7722 (RA 7722)[9] limiting the CHED's coverage to
public and private institutions of higher education as well as degree-granting programs in post-secondary educational
institutions.
CHED was given the authority to regulate and establish review centers uner EO 566
While it may be true that regulation of review centers is not one of the mandates of CHED under Republic Act
7722, however, on September 8, 2006, Her Excellency, President Gloria Macapagal-Arroyo, issued Executive Order No.
566 directing the Commission on Higher
Education to regulate the establishment and operation of review centers and similar entities in the entire country.
With the issuance of the aforesaid Executive Order, the CHED now is the agency that is mandated to regulate
the establishment and operation of all review centers as provided for under Section 4 of the Executive Order which
provides that "No review center or similar... entities shall be established and/or operate review classes without the
favorable expressed indorsement of the CHED and without the issuance of the necessary permits or authorizations to
conduct review classes. x x x"
pertinent provision of the IRR
Section 1. Authority to Establish and Operate - Only CHED recognized, accredited and reputable HEIs may be authorized
to establish and operate review center/course by the CHED upon full compliance with the conditions and requirements
provided herein and in other pertinent laws,... rules and regulations. In addition, a consortium or consortia of qualified
schools and/or entities may establish and operate review centers or conduct review classes upon compliance with the
provisions of these Rules.

Issues: The issues raised in this case are the following:


1. Whether EO 566 is an unconstitutional exercise by the Executive of legislative power as it expands the
CHED's jurisdiction; and
2. Whether the RIRR is an invalid exercise of the Executive's rule-making power.

Ruling:Main issue relating to Art. VII


As head of the Executive Department, the President is the Chief Executive. He represents the government as a
whole and sees to it that all laws are enforced by the officials and employees of his department. He has control over the
executive department, bureaus and offices. This... means that he has the authority to assume directly the functions of the
executive department, bureau and office, or interfere with the discretion of its officials. Corollary to the power of control,
the President also has the duty of supervising the enforcement of laws for the... maintenance of general peace and public
order. Thus, he is granted administrative power over bureaus and offices under his control to enable him to discharge his
duties effectively.
Administrative power is concerned with the work of applying policies and enforcing orders as determined by
proper governmental organs. It enables the President to fix a uniform standard of administrative efficiency and check the
official conduct of his agents. To this end, he... can issue administrative orders, rules and regulations.
"Sec. 3. Administrative Orders. - Acts of the President which relate to particular aspects of governmental operation in
pursuance of his duties as administrative head shall be promulgated in administrative orders."
An administrative order is an ordinance issued by the President which relates to specific aspects in the administrative
operation of government. It must be in harmony with the law and should be for the sole purpose of implementing the law
and carrying out the legislative... policy. x x x.
Since EO 566 is an invalid exercise of legislative power, the RIRR is also an invalid exercise of the CHED's
quasi-legislative power.
Administrative agencies exercise their quasi-legislative or rule-making power through the promulgation of rules and
regulations.[36] The CHED may only exercise its rule-making power within the confines of its jurisdiction under RA 7722.
The RIRR covers... review centers and similar entities which are neither institutions of higher education nor institutions
offering degree-granting programs.

Exercise of Police Power


Police power primarily rests with the legislature although it may be exercised by the President and administrative
boards by virtue of a valid delegation
Here, no delegation of police power exists under RA 7722 authorizing the President... to regulate the operations
of non-degree granting review centers.
Republic Act No. 8981 is Not the Appropriate Law
There is no doubt that a principal mandate of the PRC is to preserve the integrity of licensure examinations. The
PRC has the power to adopt measures to preserve the integrity and inviolability of licensure examinations. However, this
power should properly be interpreted to... refer to the conduct of the examinations
These powers of the PRC have nothing to do at all with the regulation of review centers.
However, this power has nothing to do with the regulation of review centers. The PRC has the power to bar PRB
members from conducting review classes in review centers. However, to... interpret this power to extend to the power to
regulate review centers is clearly an unwarranted interpretation of RA 8981.
Section 7(y) of RA 8981 giving the PRC the power to perform "such other functions and duties as may be necessary to
carry out the provisions" of RA 8981 does not extend to the regulation of review centers. There is absolutely nothing in RA
8981 that mentions regulation by the PRC of review centers.
Similarly, the PRC has no mandate to regulate similar entities whose reviewees will not even... take any
licensure examination given by the PRC.
WHEREFORE, we GRANT the petition and the petition-in-intervention. We DECLARE Executive Order No. 566
and Commission on Higher Education Memorandum Order No. 30, series of 2007 VOID for being unconstitutional. SO
ORDERED.
4. Province of North Cotabato vs GRP Peace Panel on Ancestral Domain
G.R. No. 1833591, October 14, 2008

Decision: CARPIO MORALES, J.:


Subject of these consolidated cases is the extent of the powers of the President in pursuing the peace process.
While the facts surrounding this controversy center on the armed conflict in Mindanao between the government
and the Moro Islamic Liberation Front (MILF), the legal issue involved has a bearing on all areas in the country
where there has been a long-standing armed conflict. Yet again, the Court is tasked to perform a delicate
balancing act. It must uncompromisingly delineate the bounds within which the President may lawfully exercise
her discretion, but it must do so in strict adherence to the Constitution, lest its ruling unduly restricts the freedom
of action vested by that same Constitution in the Chief Executive precisely to enable her to pursue the peace
process effectively.

Facts: On August 5, 2008, the Government of the Republic of the Philippines (GRP) and the MILF, through the
Chairpersons of their respective peace negotiating panels, were scheduled to sign a Memorandum of Agreement
on the Ancestral Domain (MOA-AD) Aspect of the GRP-MILF Tripoli Agreement on Peace of 2001 in Kuala
Lumpur, Malaysia.
The signing of the MOA-AD between the GRP and the MILF was not to materialize, however, for upon
motion of petitioners, specifically those who filed their cases before the scheduled signing of the MOA-AD, this
Court issued a Temporary Restraining Order enjoining the GRP from signing the same.
The MOA-AD was preceded by a long process of negotiation and the concluding of several prior
agreements between the two parties beginning in 1996, when the GRP-MILF peace negotiations began. On July
18, 1997, the GRP and MILF Peace Panels signed the Agreement on General Cessation of Hostilities. The
following year, they signed the General Framework of Agreement of Intent on August 27, 1998.
On July 23, 2008, the Province of North Cotabato and Vice-Governor Emmanuel Piñol filed a petition,
docketed as G.R. No. 183591, for Mandamus and Prohibition with Prayer for the Issuance of Writ of Preliminary
Injunction and Temporary Restraining Order. Invoking the right to information on matters of public concern,
petitioners seek to compel respondents to disclose and furnish them the complete and official copies of the MOA-
AD including its attachments, and to prohibit the slated signing of the MOA-AD, pending the disclosure of the
contents of the MOA-AD and the holding of a public consultation thereon. Supplementarily, petitioners pray that
the MOA-AD be declared unconstitutional.

Issues: 1. Whether the petitions have become moot and academic


(i) insofar as the mandamus aspect is concerned, in view of the disclosure of official copies of the final draft of
the Memorandum of Agreement (MOA); and
(ii) insofar as the prohibition aspect involving the Local Government Units is concerned, if it is considered that
consultation has become fait accompli with the finalization of the draft;
2. Whether the constitutionality and the legality of the MOA is ripe for adjudication;
3. Whether respondent Government of the Republic of the Philippines Peace Panel committed grave
abuse of discretion amounting to lack or excess of jurisdiction when it negotiated and initiated the MOA vis-à-vis
ISSUES Nos. 4 and 5;
4. Whether there is a violation of the people's right to information on matters of public concern (1987
Constitution, Article III, Sec. 7) under a state policy of full disclosure of all its transactions involving public interest
(1987 Constitution, Article II, Sec. 28) including public consultation under Republic Act No. 7160 (LOCAL
GOVERNMENT CODE OF 1991)[;] If it is in the affirmative, whether prohibition under Rule 65 of the 1997 Rules
of Civil Procedure is an appropriate remedy;
5. Whether by signing the MOA, the Government of the Republic of the Philippines would be BINDING
itself
a) to create and recognize the Bangsamoro Juridical Entity (BJE) as a separate state, or a juridical, territorial or
political subdivision not recognized by law;
b) to revise or amend the Constitution and existing laws to conform to the MOA;
c) to concede to or recognize the claim of the Moro Islamic Liberation Front for ancestral domain in violation of
Republic Act No. 8371 (THE INDIGENOUS PEOPLES RIGHTS ACT OF 1997), particularly Section 3(g) &
Chapter VII (DELINEATION, RECOGNITION OF ANCESTRAL DOMAINS)[;]
If in the affirmative, whether the Executive Branch has the authority to so bind the Government of the
Republic of the Philippines;
6. Whether the inclusion/exclusion of the Province of North Cotabato, Cities of Zamboanga, Iligan and
Isabela, and the Municipality of Linamon, Lanao del Norte in/from the areas covered by the projected
Bangsamoro Homeland is a justiciable question; and
7. Whether desistance from signing the MOA derogates any prior valid commitments of the Government
of the Republic of the Philippines.

Held: The main body of the MOA-AD is divided into four strands, namely, Concepts and Principles, Territory,
Resources, and Governance.
The power of judicial review is limited to actual cases or controversies. Courts decline to issue advisory
opinions or to resolve hypothetical or feigned problems, or mere academic questions. The limitation of the power
of judicial review to actual cases and controversies defines the role assigned to the judiciary in a tripartite
allocation of power, to assure that the courts will not intrude into areas committed to the other branches of
government.
As the petitions involve constitutional issues which are of paramount public interest or of transcendental
importance, the Court grants the petitioners, petitioners-in-intervention and intervening respondents the requisite
locus standi in keeping with the liberal stance adopted in David v. Macapagal-Arroyo.
Contrary to the assertion of respondents that the non-signing of the MOA-AD and the eventual
dissolution of the GRP Peace Panel mooted the present petitions, the Court finds that the present petitions
provide an exception to the "moot and academic" principle in view of (a) the grave violation of the Constitution
involved; (b) the exceptional character of the situation and paramount public interest; (c) the need to formulate
controlling principles to guide the bench, the bar, and the public; and (d) the fact that the case is capable of
repetition yet evading review.
The MOA-AD is a significant part of a series of agreements necessary to carry out the GRP-MILF Tripoli
Agreement on Peace signed by the government and the MILF back in June 2001. Hence, the present MOA-AD
can be renegotiated or another one drawn up that could contain similar or significantly dissimilar provisions
compared to the original.
That the subject of the information sought in the present cases is a matter of public concern faces no
serious challenge. In fact, respondents admit that the MOA-AD is indeed of public concern. In previous cases,
the Court found that the regularity of real estate transactions entered in the Register of Deeds, the need for
adequate notice to the public of the various laws, the civil service eligibility of a public employee, the proper
management of GSIS funds allegedly used to grant loans to public officials, the recovery of the Marcoses'
alleged ill-gotten wealth, and the identity of party-list nominees, among others, are matters of public concern.
Undoubtedly, the MOA-AD subject of the present cases is of public concern, involving as it does the sovereignty
and territorial integrity of the State, which directly affects the lives of the public at large.
In sum, the Presidential Adviser on the Peace Process committed grave abuse of discretion when he
failed to carry out the pertinent consultation process, as mandated by E.O. No. 3, Republic Act No. 7160, and
Republic Act No. 8371. The furtive process by which the MOA-AD was designed and crafted runs contrary to and
in excess of the legal authority, and amounts to a whimsical, capricious, oppressive, arbitrary and despotic
exercise thereof. It illustrates a gross evasion of positive duty and a virtual refusal to perform the duty enjoined.
The MOA-AD cannot be reconciled with the present Constitution and laws. Not only its specific
provisions but the very concept underlying them, namely, the associative relationship envisioned between the
GRP and the BJE, are unconstitutional, for the concept presupposes that the associated entity is a state and
implies that the same is on its way to independence. The Memorandum of Agreement on the Ancestral Domain
Aspect of the GRP-MILF Tripoli Agreement on Peace of 2001 is declared contrary to law and the Constitution.

5. BIRAOGO VS PTC
G.R. No. 192935 December 7, 2010
LOUIS “BAROK” C. BIRAOGO
vs.
THE PHILIPPINE TRUTH COMMISSION OF 2010
x – – – – – – – – – – – – – – – – – – – – – – -x
G.R. No. 193036
REP. EDCEL C. LAGMAN, REP. RODOLFO B. ALBANO, JR., REP. SIMEON A. DATUMANONG, and REP.
ORLANDO B. FUA, SR.
vs.
EXECUTIVE SECRETARY PAQUITO N. OCHOA, JR. and DEPARTMENT OF BUDGET AND MANAGEMENT
SECRETARY FLORENCIO B. ABAD

FACTS: Pres. Aquino signed E. O. No. 1 establishing Philippine Truth Commission of 2010 (PTC) dated July 30,
2010.
PTC is a mere ad hoc body formed under the Office of the President with the primary task to investigate
reports of graft and corruption committed by third-level public officers and employees, their co-principals,
accomplices and accessories during the previous administration, and to submit its finding and recommendations
to the President, Congress and the Ombudsman. PTC has all the powers of an investigative body. But it is not a
quasi-judicial body as it cannot adjudicate, arbitrate, resolve, settle, or render awards in disputes between
contending parties. All it can do is gather, collect and assess evidence of graft and corruption and make
recommendations. It may have subpoena powers but it has no power to cite people in contempt, much less order
their arrest. Although it is a fact-finding body, it cannot determine from such facts if probable cause exists as to
warrant the filing of an information in our courts of law.
Petitioners asked the Court to declare it unconstitutional and to enjoin the PTC from performing its
functions. They argued that:
(a) E.O. No. 1 violates separation of powers as it arrogates the power of the Congress to create a public
office and appropriate funds for its operation.
(b) The provision of Book III, Chapter 10, Section 31 of the Administrative Code of 1987 cannot
legitimize E.O. No. 1 because the delegated authority of the President to structurally reorganize the Office of the
President to achieve economy, simplicity and efficiency does not include the power to create an entirely new
public office which was hitherto inexistent like the “Truth Commission.”
(c) E.O. No. 1 illegally amended the Constitution and statutes when it vested the “Truth Commission”
with quasi-judicial powers duplicating, if not superseding, those of the Office of the Ombudsman created under
the 1987 Constitution and the DOJ created under the Administrative Code of 1987.
(d) E.O. No. 1 violates the equal protection clause as it selectively targets for investigation and
prosecution officials and personnel of the previous administration as if corruption is their peculiar species even as
it excludes those of the other administrations, past and present, who may be indictable.
Respondents, through OSG, questioned the legal standing of petitioners and argued that:
1] E.O. No. 1 does not arrogate the powers of Congress because the President’s executive power and
power of control necessarily include the inherent power to conduct investigations to ensure that laws are faithfully
executed and that, in any event, the Constitution, Revised Administrative Code of 1987, PD No. 141616 (as
amended), R.A. No. 9970 and settled jurisprudence, authorize the President to create or form such bodies.
2] E.O. No. 1 does not usurp the power of Congress to appropriate funds because there is no
appropriation but a mere allocation of funds already appropriated by Congress.
3] The Truth Commission does not duplicate or supersede the functions of the Ombudsman and the
DOJ, because it is a fact-finding body and not a quasi-judicial body and its functions do not duplicate, supplant or
erode the latter’s jurisdiction.
4] The Truth Commission does not violate the equal protection clause because it was validly created for
laudable purposes.

ISSUES: 1. WON the petitioners have legal standing to file the petitions and question E. O. No. 1;
2. WON E. O. No. 1 violates the principle of separation of powers by usurping the powers of Congress
to create and to appropriate funds for public offices, agencies and commissions;
3. WON E. O. No. 1 supplants the powers of the Ombudsman and the DOJ;
4. WON E. O. No. 1 violates the equal protection clause.
RULING:

The power of judicial review is subject to limitations, to wit: (1) there must be an actual case or controversy calling for
the exercise of judicial power; (2) the person challenging the act must have the standing to question the validity of the
subject act or issuance; otherwise stated, he must have a personal and substantial interest in the case such that he has
sustained, or will sustain, direct injury as a result of its enforcement; (3) the question of constitutionality must be raised at
the earliest opportunity; and (4) the issue of constitutionality must be the very lis mota of the case.
1. The petition primarily invokes usurpation of the power of the Congress as a body to which they belong as
members. To the extent the powers of Congress are impaired, so is the power of each member thereof, since his
office confers a right to participate in the exercise of the powers of that institution.
Legislators have a legal standing to see to it that the prerogative, powers and privileges vested by the
Constitution in their office remain inviolate. Thus, they are allowed to question the validity of any official action
which, to their mind, infringes on their prerogatives as legislators.
With regard to Biraogo, he has not shown that he sustained, or is in danger of sustaining, any personal
and direct injury attributable to the implementation of E. O. No. 1.
Locus standi is “a right of appearance in a court of justice on a given question.” In private suits, standing
is governed by the “real-parties-in interest” rule. It provides that “every action must be prosecuted or defended in
the name of the real party in interest.” Real-party-in interest is “the party who stands to be benefited or injured by
the judgment in the suit or the party entitled to the avails of the suit.”
Difficulty of determining locus standi arises in public suits. Here, the plaintiff who asserts a “public right”
in assailing an allegedly illegal official action, does so as a representative of the general public. He has to show
that he is entitled to seek judicial protection. He has to make out a sufficient interest in the vindication of the
public order and the securing of relief as a “citizen” or “taxpayer.
The person who impugns the validity of a statute must have “a personal and substantial interest in the
case such that he has sustained, or will sustain direct injury as a result.” The Court, however, finds reason in
Biraogo’s assertion that the petition covers matters of transcendental importance to justify the exercise of
jurisdiction by the Court. There are constitutional issues in the petition which deserve the attention of this Court in
view of their seriousness, novelty and weight as precedents
The Executive is given much leeway in ensuring that our laws are faithfully executed. The powers of the
President are not limited to those specific powers under the Constitution. One of the recognized powers of the
President granted pursuant to this constitutionally-mandated duty is the power to create ad hoc committees. This
flows from the obvious need to ascertain facts and determine if laws have been faithfully executed. The purpose
of allowing ad hoc investigating bodies to exist is to allow an inquiry into matters which the President is entitled to
know so that he can be properly advised and guided in the performance of his duties relative to the execution
and enforcement of the laws of the land.

2. There will be no appropriation but only an allotment or allocations of existing funds already appropriated. There is
no usurpation on the part of the Executive of the power of Congress to appropriate funds. There is no need to specify the
amount to be earmarked for the operation of the commission because, whatever funds the Congress has provided for the
Office of the President will be the very source of the funds for the commission. The amount that would be allocated to the
PTC shall be subject to existing auditing rules and regulations so there is no impropriety in the funding.

3. PTC will not supplant the Ombudsman or the DOJ or erode their respective powers. If at all, the investigative
function of the commission will complement those of the two offices. The function of determining probable cause for the
filing of the appropriate complaints before the courts remains to be with the DOJ and the Ombudsman. PTC’s power to
investigate is limited to obtaining facts so that it can advise and guide the President in the performance of his duties
relative to the execution and enforcement of the laws of the land.

4. Court finds difficulty in upholding the constitutionality of Executive Order No. 1 in view of its apparent transgression
of the equal protection clause enshrined in Section 1, Article III (Bill of Rights) of the 1987 Constitution.
Equal protection requires that all persons or things similarly situated should be treated alike, both as to rights
conferred and responsibilities imposed. It requires public bodies and institutions to treat similarly situated individuals in a
similar manner. The purpose of the equal protection clause is to secure every person within a state’s jurisdiction against
intentional and arbitrary discrimination, whether occasioned by the express terms of a statue or by its improper execution
through the state’s duly constituted authorities.
There must be equality among equals as determined according to a valid classification. Equal protection clause
permits classification. Such classification, however, to be valid must pass the test of reasonableness. The test has four
requisites: (1) The classification rests on substantial distinctions; (2) It is germane to the purpose of the law; (3) It is not
limited to existing conditions only; and (4) It applies equally to all members of the same class.
The classification will be regarded as invalid if all the members of the class are not similarly treated,
both as to rights conferred and obligations imposed.
Executive Order No. 1 should be struck down as violative of the equal protection clause. The clear
mandate of truth commission is to investigate and find out the truth concerning the reported cases of graft and
corruption during the previous administration only. The intent to single out the previous administration is plain,
patent and manifest.
Arroyo administration is but just a member of a class, that is, a class of past administrations. It is not a
class of its own. Not to include past administrations similarly situated constitutes arbitrariness which the equal
protection clause cannot sanction. Such discriminating differentiation clearly reverberates to label the
commission as a vehicle for vindictiveness and selective retribution. Superficial differences do not make for a
valid classification.
The PTC must not exclude the other past administrations. The PTC must, at least, have the authority to
investigate all past administrations.
The Constitution is the fundamental and paramount law of the nation to which all other laws must
conform and in accordance with which all private rights determined and all public authority administered. Laws
that do not conform to the Constitution should be stricken down for being unconstitutional.
WHEREFORE, the petitions are GRANTED. Executive Order No. 1 is hereby declared
UNCONSTITUTIONAL insofar as it is violative of the equal protection clause of the Constitution.

SECTION 13

6. Funa vs Agra
G.R. No. 191644 February 19, 2013

Facts: The petitioner alleges that on March 1, 2010, President Gloria M. Macapagal Arroyo appointed Agra as the
Acting Secretary of Justice following the resignation of Secretary Agnes VST Devanadera in order to vie for a
congressional seat in Quezon Province; that on March 5, 2010, President Arroyo designated Agra as the Acting Solicitor
General in a concurrent capacity; that on April 7, 2010, the petitioner, in his capacity as a taxpayer, a concerned citizen
and a lawyer, commenced this suit to challenge the constitutionality of Agra’s concurrent appointments or designations,
claiming it to be prohibited under Section 13, Article VII of the 1987 Constitution; that during the pendency of the suit,
President Benigno S. Aquino III appointed Atty. Jose Anselmo I. Cadiz as the Solicitor General; and that Cadiz
assumed as the Solicitor General and commenced his duties as such on August 5, 2010. Agra renders a different version
of the antecedents. He represents that on January 12, 2010, he was then the Government Corporate Counsel when
President Arroyo designated him as the Acting Solicitor General in place of Solicitor General Devanadera who had been
appointed as the Secretary of Justice; that on March 5, 2010, President Arroyo designated him also as the Acting
Secretary of Justice vice Secretary Devanadera who had meanwhile tendered her resignation in order to run for Congress
representing a district in Quezon Province in the May 2010 elections; that he then relinquished his position as the
Government Corporate Counsel; and that pending the appointment of his successor, Agra continued to perform his duties
as the Acting Solicitor General.
Notwithstanding the conflict in the versions of the parties, the fact that Agra has admitted to holding the two
offices concurrently in acting capacities is settled, which is sufficient for purposes of resolving the constitutional question
that petitioner raises herein.

Issue: Whether or not Agra’s holding of concurrent position is unconstitutional.

Held: Yes. At the center of the controversy is the correct application of Section 13, Article VII of the 1987 Constitution,
viz:
Section 13. The President, Vice-President, the Members of the Cabinet, and their deputies or assistants shall not, unless
otherwise provided in this Constitution, hold any other office or employment during their tenure. They shall not, during said
tenure, directly or indirectly practice any other profession, participate in any business, or be financially interested in any
contract with, or in any franchise, or special privilege granted by the Government or any subdivision, agency, or
instrumentality thereof, including government-owned or controlled corporations or their subsidiaries. They shall strictly
avoid conflict of interest in the conduct of their office.

A relevant and complementing provision is Section 7, paragraph (2), Article IX-B of the 1987 Constitution, to wit:
Section 7. x x x Unless otherwise allowed by law or the primary functions of his position, no appointive official shall hold
any other office or employment in the Government or any subdivision, agency or instrumentality thereof, including
government-owned or controlled corporations or their subsidiaries.
Being designated as the Acting Secretary of Justice concurrently with his position of Acting Solicitor General,
therefore, Agra was undoubtedly covered by Section 13, Article VII, supra, whose text and spirit were too clear to be
differently read. Hence, Agra could not validly hold any other office or employment during his tenure as the Acting Solicitor
General, because the Constitution has not otherwise so provided.
It was of no moment that Agra’s designation was in an acting or temporary capacity. The text of Section 13,
supra, plainly indicates that the intent of the Framers of the Constitution was to impose a stricter prohibition on the
President and the Members of his Cabinet in so far as holding other offices or employments in the Government or in
government-owned or government controlled-corporations was concerned. In this regard, to hold an office means to
possess or to occupy the office, or to be in possession and administration of the office, which implies nothing less than the
actual discharge of the functions and duties of the office. Indeed, in the language of Section 13 itself, supra, the
Constitution makes no reference to the nature of the appointment or designation. The prohibition against dual or multiple
offices being held by one official must be construed as to apply to all appointments or designations, whether permanent or
temporary, for it is without question that the avowed objective of Section 13, supra, is to prevent the concentration of
powers in the Executive Department officials, specifically the President, the Vice-President, the Members of the Cabinet
and their deputies and assistants.
To construe differently is to “open the veritable floodgates of circumvention of an important constitutional
disqualification of officials in the Executive Department and of limitations on the Presidents power of appointment in the
guise of temporary designations of Cabinet Members, undersecretaries and assistant secretaries as officers-in-charge of
government agencies, instrumentalities, or government-owned or controlled corporations.
It is not amiss to observe, lastly, that assuming that Agra, as the Acting Solicitor General, was not covered by the
stricter prohibition under Section 13, supra, due to such position being merely vested with a cabinet rank under Section 3,
Republic Act No. 9417, he nonetheless remained covered by the general prohibition under Section 7, supra. Hence, his
concurrent designations were still subject to the conditions under the latter constitutional provision. In this regard, the
Court aptly pointed out in Public Interest Center, Inc. v. Elma:
The general rule contained in Article IX-B of the 1987 Constitution permits an appointive official to hold more than
one office only if “allowed by law or by the primary functions of his position.” In the case of Quimson v. Ozaeta, this Court
ruled that, “[t]here is no legal objection to a government official occupying two government offices and performing the
functions of both as long as there is no incompatibility.” The crucial test in determining whether incompatibility exists
between two offices was laid out in People v. Green – whether one office is subordinate to the other, in the sense that one
office has the right to interfere with the other.

9. Funa vs Executive Secretary with Notes


G.R. No. 184740 February 11, 2010
DENNIS A. B. FUNA, Petitioner, vs. EXECUTIVE SECRETARY EDUARDO R. ERMITA, Office of the President, SEC.
LEANDRO R. MENDOZA, in his official capacity as Secretary of the Department of Transportation and
Communications, USEC. MARIA ELENA H. BAUTISTA, in her official capacities as Undersecretary of the
Department of Transportation and Communications and as Officer-in-Charge of the Maritime Industry Authority
(MARINA), Respondents.
DECISION
VILLARAMA, JR., J.:

Facts:
This is a petition for certiorari, prohibition and mandamus under Rule 65 with prayer for the issuance of a
temporary restraining order and/or writ of preliminary injunction, to declare as unconstitutional the designation of
respondent Undersecretary Maria Elena H. Bautista as Officer-in-Charge (OIC) of the Maritime Industry Authority
(MARINA).
On October 4, 2006, President Gloria Macapagal-Arroyo appointed respondent Maria Elena H. Bautista
(Bautista) as Undersecretary of the Department of Transportation and Communications (DOTC).
On September 1, 2008, following the resignation of then MARINA Administrator Vicente T. Suazo, Jr., Bautista was
designated as Officer-in-Charge (OIC), Office of the Administrator, MARINA, in concurrent capacity as DOTC
Undersecretary.
On October 21, 2008, Dennis A. B. Funa in his capacity as taxpayer, concerned citizen and lawyer, filed the
instant petition challenging the constitutionality of Bautista’s appointment/designation, which is proscribed by the
prohibition on the President, Vice-President, the Members of the Cabinet, and their deputies and assistants to hold any
other office or employment.
On January 5, 2009, during the pendency of this petition, Bautista was appointed Administrator of the
MARINA and she assumed her duties and responsibilities as such on February 2, 2009.
Petitioner argues that Bautista’s concurrent positions as DOTC Undersecretary and MARINA OIC is in violation of Section
13, Article VII of the 1987 Constitution .
On the other hand, the respondents argue that the requisites of a judicial inquiry are not present in this case. In
fact, there no longer exists an actual controversy that needs to be resolved in view of the appointment of respondent
Bautista as MARINA Administrator effective February 2, 2009 and the relinquishment of her post as DOTC
Undersecretary for Maritime Transport, which rendered the present petition moot and academic. Petitioner’s prayer for a
temporary restraining order or writ of preliminary injunction is likewise moot and academic since, with this supervening
event, there is nothing left to enjoin.

Issue: Whether or not the designation of respondent Bautista as OIC of MARINA, concurrent with the position of DOTC
Undersecretary for Maritime Transport to which she had been appointed, violated the constitutional proscription against
dual or multiple offices for Cabinet Members and their deputies and assistants.

Held: The petition is meritorious.


Petitioner having alleged a grave violation of the constitutional prohibition against Members of the Cabinet, their
deputies and assistants holding two (2) or more positions in government, the fact that he filed this suit as a concerned
citizen sufficiently confers him with standing to sue for redress of such illegal act by public officials.
A moot and academic case is one that ceases to present a justiciable controversy by virtue of supervening
events, so that a declaration thereon would be of no practical use or value. Generally, courts decline jurisdiction over such
case or dismiss it on ground of mootness. But even in cases where supervening events had made the cases moot, this
Court did not hesitate to resolve the legal or constitutional issues raised to formulate controlling principles to guide the
bench, bar, and public. In the present case, the mootness of the petition does not bar its resolution.
Resolution of the present controversy hinges on the correct application of Section 13, Article VII of the 1987 Constitution,
which provides:
Sec. 13. The President, Vice-President, the Members of the Cabinet, and their deputies or assistants shall not,
unless otherwise provided in this Constitution, hold any other office or employment during their tenure. They
shall not, during said tenure, directly or indirectly practice any other profession, participate in any business, or be
financially interested in any contract with, or in any franchise, or special privilege granted by the Government or any
subdivision, agency, or instrumentality thereof, including government-owned or controlled corporations or their
subsidiaries. They shall strictly avoid conflict of interest in the conduct of their office.
The spouse and relatives by consanguinity or affinity within the fourth civil degree of the President shall not, during his
tenure, be appointed as Members of the Constitutional Commissions, or the Office of the Ombudsman, or as Secretaries,
Undersecretaries, chairmen or heads of bureaus or offices, including government-owned or controlled corporations and
their subsidiaries.
On the other hand, Section 7, paragraph (2), Article IX-B reads:
Sec. 7. x x x
Unless otherwise allowed by law or the primary functions of his position, no appointive official shall hold any other office or
employment in the Government or any subdivision, agency or instrumentality thereof, including government-owned or
controlled corporations or their subsidiaries.
Noting that the prohibition imposed on the President and his official family is all-embracing, the disqualification
was held to be absolute, as the holding of "any other office" is not qualified by the phrase "in the Government"
unlike in Section 13, Article VI prohibiting Senators and Members of the House of Representatives from holding
"any other office or employment in the Government"; and when compared with other officials and employees such as
members of the armed forces and civil service employees, we concluded thus:
These sweeping, all-embracing prohibitions imposed on the President and his official family, which prohibitions
are not similarly imposed on other public officials or employees such as the Members of Congress, members of the
civil service in general and members of the armed forces, are proof of the intent of the 1987 Constitution to treat the
President and his official family as a class by itself and to impose upon said class stricter prohibitions.
Thus, while all other appointive officials in the civil service are allowed to hold other office or employment in the
government during their tenure when such is allowed by law or by the primary functions of their positions, members of the
Cabinet, their deputies and assistants may do so only when expressly authorized by the Constitution itself. In other words,
Section 7, Article IX-B is meant to lay down the general rule applicable to all elective and appointive public
officials and employees, while Section 13, Article VII is meant to be the exception applicable only to the
President, the Vice-President, Members of the Cabinet, their deputies and assistants.
Since the evident purpose of the framers of the 1987 Constitution is to impose a stricter prohibition on the
President, Vice-President, members of the Cabinet, their deputies and assistants with respect to holding multiple offices
or employment in the government during their tenure, the exception to this prohibition must be read with equal severity.
On its face, the language of Section 13, Article VII is prohibitory so that it must be understood as intended to be a positive
and unequivocal negation of the privilege of holding multiple government offices or employment. Verily, wherever the
language used in the constitution is prohibitory, it is to be understood as intended to be a positive and unequivocal
negation. The phrase "unless otherwise provided in this Constitution" must be given a literal interpretation to
refer only to those particular instances cited in the Constitution itself, to wit: the Vice-President being appointed
as a member of the Cabinet under Section 3, par. (2), Article VII; or acting as President in those instances
provided under Section 7, pars. (2) and (3), Article VII; and, the Secretary of Justice being ex-officio member of
the Judicial and Bar Council by virtue of Section 8 (1), Article VIII.
Respondent Bautista being then the appointed Undersecretary of DOTC, she was thus covered by the stricter
prohibition under Section 13, Article VII and consequently she cannot invoke the exception provided in Section 7,
paragraph 2, Article IX-B where holding another office is allowed by law or the primary functions of the position. Neither
was she designated OIC of MARINA in an ex-officio capacity, which is the exception recognized in Civil Liberties Union.
WHEREFORE, the petition is GRANTED. The designation of respondent Ma. Elena H. Bautista as Officer-in-
Charge, Office of the Administrator, Maritime Industry Authority, in a concurrent capacity with her position as DOTC
Undersecretary for Maritime Transport, is hereby declared UNCONSTITUTIONAL for being violative of Section 13, Article
VII of the 1987 Constitution and therefore, NULL and VOID.

7. CIVIL LIBERTIES UNION VS. EXECUTIVE SECRETARY


(194 SCRA 317)

FACTS: President Aquino issued EO No. 284, which allows members of the Cabinet, their undersecretaries and assistant
secretaries to hold other government offices or positions in addition to their primary positions. It was assailed for it violates
the Constitution.
Petitioners challenge the constitutionality of EO No. 284 on the principal submission that it adds exceptions to
Section 13, Article VII other than those provided in the Constitution. According to petitioners, by virtue of the phrase
"unless otherwise provided in this Constitution," the only exceptions against holding any other office or employment in
Government are those provided in the Constitution, namely: (1) The Vice-President may be appointed as a Member of the
Cabinet under Section 3, par. (2), Article VII thereof; and (2) the Secretary of Justice is an ex-officio member of the
Judicial and Bar Council by virtue of Section 8 (1), Article VIII.

ISSUE: Whether or not an executive order allowing members of the Cabinet, their undersecretaries and assistant
secretaries to hold other government offices in addition to their primary positions is valid.

HELD: Invalid. In the light of the construction given to Section 13, Article VII in relation to Section 7, par. (2), Article IX-B
of the 1987 Constitution, Executive Order No. 284 dated July 23, 1987 is unconstitutional. Ostensibly restricting the
number of positions that Cabinet members, undersecretaries or assistant secretaries may hold in addition to their primary
position to not more than two (2) positions in the government and government corporations, Executive Order No. 284
actually allows them to hold multiple offices or employment in direct contravention of the express mandate of Section 13,
Article VII of the 1987 Constitution prohibiting them from doing so, unless otherwise provided in the 1987 Constitution
itself.

8. DENNIS FUNA V EXEC SECRETARY


9. AYTONA V CASTILLO
4 SCRA 1 – Political Law – Appointing Power – Midnight Appointments
Dominador Aytona was one of those appointed by outgoing president Carlos Garcia during the last day of his
term. Aytona was appointed as the ad interim governor of the Central Bank. When the next president, Diosdado
Macapagal took his office, he issued Order No. 2 which recalled Aytona’s position and at the same time he appointed
Andres Castillo as the new governor of the Central Bank. Aytona then filed a quo warranto proceeding claiming that
he is qualified to remain as the Central Bank governor and that he was validly appointed by the former president.
Macapagal averred that the ex-president’s appointments were scandalous, irregular, hurriedly done, contrary to law
and the spirit of which, and it was an attempt to subvert the incoming presidency or administration.
ISSUE: Whether or not Aytona should remain in his post.
HELD: No. Had the appointment of Aytona been done in good faith then he would have the right to continue office.
Here, even though Aytona is qualified to remain in his post as he is competent enough, his appointment can
nevertheless be revoked by the president. Garcia’s appointments are hurried maneuvers to subvert the upcoming
administration and is set to obstruct the policies of the next president. As a general rule, once a person is qualified his
appointment should not be revoked but in here it may be since his appointment was grounded on bad faith, immorality
and impropriety. In public service, it is not only legality that is considered but also justice, fairness and righteousness.

10. IN RE APPOINTMENTS DATED MARCH 30,1998 OF HON. M. A. VALENZUELA AND HON. P. B. VILLARTA
AS RTC JUDGES
(298 SCRA 408 [NOVEMBER 9, 1998])
NARVASA, C.J.:

FACTS: Hon. Mateo A. Valenzuela and Hon. Placido B. Vallarta were appointed by the then President on March 30, 1998
as Judges of the RTC, Branch 62, Bago City and of Branch 24, Cabanatuan City, respectively. On May 12, 1998, the
Chief Justice received from Malacanang the appointments of two (2) Judge of the RTC mentioned above. The
appointments were questioned on the view that they were made during the period of the ban on appointments imposed by
Section 15, Article VII of the Constitution.
The issue was already ventilated at the meeting of the Judicial and Bar Council on March 9, 1998 regarding the
constitutionality of appointments of eight (8) Associate Justices to the Court of Appeals, specifically, in light of the
forthcoming presidential elections. Attention was drawn to Section 15, Article VII of the Constitution reading as follows:
Sec. 15. Two months immediately before the next presidential elections and up to the end of his, term, a
President or Acting President shall not make appointments, except temporary appointments to executive positions when
continued vacancies therein will prejudice public service or endanger public safety.
On the other hand, appointments to fill vacancies in the Supreme Court during the period mentioned in the provision just
quoted could be justified by Section 4(1) of Article VII of the Constitution which states:
Sec. 4 (1) The Supreme Court shall be composed of a Chief Justice and fourteen Associate Justices. ** **. Any
vacancy shall be filled within ninety days from the occurrence thereof.
Also pertinent although not specifically discussed is Section 9 of the same Article VIII which provides that for the lower
courts, the President shall issue the appointments — from a list of at least three nominees prepared by the Council for
every vacancy — within ninety days from the submission of the list.
The Court issued a Resolution which states that ‖pending the foregoing proceedings and the deliberation by the
Court on the mater, and until further orders, no action be taken on the appointments of Hon. Valenzuela and Hon. Vallarta
which in the meantime shall be held in abeyance.
In compliance with the foregoing Resolution, Hon. Valenzuela and Hon. Vallarta filed before the Court the required
pleadings and other documents. It is noted that Hon. Valenzuela had already taken his Oath of Office a Judge on May 14,
1998 and explained that he did so because on May 7, 1998, he ―received from Malacanang copy of his appointment ***
which contained the following direction ―By virtue hereof, you may qualify and enter upon the performance of the duties
of the office.
The Court then deliberated on the pleadings and documents above mentioned, in relation to the facts and
circumstances on record and thereafter resolved to promulgate the following opinion.

ISSUE: Whether, during the period of the ban on appointments imposed by Section 15, Article VII of the Constitution, the
President is nonetheless required to fill vacancies in the judiciary, in view of Sections 4(1) and 9 of Article VIII.
Whether the President can make appointments to the judiciary during the period of the ban in the interest of public
service.

HELD:
NO. The Court's view is that during the period stated in Section 15. Article VII of the Constitution — "(t)wo
months immediatey before the next presidential elections and up to the end his term" — the President is neither required
to make appointments to the courts nor allowed to do so; and that Sections 4(1) and 9 of Article VIII simply mean that the
President is required to fill vacancies in the courts within the time frames provided therein unless prohibited by Section 15
of Article VII. It is not noteworthy that the prohibition on appointments comes into effect only once every six years.
In this connection, it may be pointed out that that instruction that any "vacany shall be filled within ninety days" (in the last
sentence of Section 4 (1) of Article VIII) contrasts with the prohibition Section 15, Article VII, which is couched in stronger
negative language — that "a President or Acting President shall not make appointments. . ."
Now, it appears that Section 15, Article VI is directed against two types of appointments: (1) those made for
buying votes and (2) those made for partisan considerations. The first refers to those appointments made within the two
months preceding a Presidential election and are similar to those which are declared elections offenses in the Omnibus
Election Code. The second type of appointments prohibited by Section 15, Article VII consist of the so-called "midnight"
appointments.
Considering the respective reasons for the time frames for filling vacancies in the courts and the restriction on the
President's power of appointments, it is this Court's view that, as a general proposition, in case of conflict, the former
should yield to the latter. Surely, the prevention of vote-buying and similar evils outweighs the need for avoiding delays in
filling up of court vacancies or the disposition of some cases. Temporary vacancies can abide the period of the ban which,
incidentally and as earlier pointed out, comes to exist only once in every six years. Moreover, those occurring in the lower
courts can be filled temporarily by designation. But prohibited appointments are long-lasting and permanent in their
effects. They may, as earlier pointed out, their making is considered an election offense.

To be sure, instances may be conceived of the imperative need for an appointment, during the period of the ban,
not only in the executive but also in the Supreme Court. This may be the case should the membership of the Court be so
reduced that it will have no quorum, or should the voting on a particularly important question requiring expeditious
resolution be evenly divided. Such a case, however, is covered by neither Section 15 of Article VII nor Sections 4 (1) and
9 of Article VIII.
Concerning Valenzuela‘s oath-taking and ―reporting for duty‖ as Presiding Judge of RTC Branch 62, Bago City,
on May 14, 1998, it must be noted that it is a standing practice on the appointments to the Judiciary – from the highest to
the lowest court – to be sent by the Office of the President to the Office of the Chief Justice, the appointments being
addressed to the appointees. It is the Clerk of Court of the Supreme Court in the Chief‘s Justice behalf, who thereafter
advises the individual appointees of their appointments and also the date commencement of the pre-requisite orientation
seminar to be conducted by the Philippine Judicial Academy for new Judges. The procedure ensures the authenticity of
the appointments, enables the Court, particularly the Office of the Court Administrator, to enter in the appropriate records
all appointments to the Judiciary a well as other relevant data such as the dates of qualification, the completion by the
appointees of their pre-requisite orientation seminars, their assumption of duty, etc. The procedure also precludes the
possibility, however remote of Judges acting on spurious or otherwise defective appointments.
The appointments of Messrs. Valenzuela and Vallarta on March 30, 1998 (transmitted to the Office of the Chief
Justice on May 14, 998) were unquestionably made during the period of the ban. Consequently, they come within the
operation of the first prohibition relating to appointments which are considered to be for the purpose of buying votes or
influencing the election. While the filling of vacancies in the judiciary is undoubtedly in the public interest, there is no
showing in this case of any compelling reason to justify the making of the appointments during the period of the ban. On
the other hand, as already discussed, there is a strong public policy for the prohibition against appointments made within
the period of the ban.
In view of the foregoing considerations, the Court Resolved to DECLARE VOID the appointments signed by His
Excellency the President under date of March 30, 1998 of Hon. Mateo A. Valenzuela and Hon. Placido B. Vallarta as
Judges of the Regional Trial Court of Branch 62, Bago City and of Branch 24, Cabanatuan City, respectively and to order
them, forthwith on being served with notice of this decision, to forthwith CEASE AND DESIST from discharging the office
of Judge of the Courts to which they were respectively appointed on March 30, 1998.
11. ARTURO DE CASTRO V JBC
ARTURO M. DE CASTRO vs. JUDICIAL AND BAR COUNCIL (JBC) and PRESIDENT GLORIA MACAPAGAL –
ARROYO
G.R. No. 191002, March 17, 2010
FACTS: The compulsory retirement of Chief Justice Reynato S. Puno by May 17, 2010 occurs just days after the coming
presidential elections on May 10, 2010.
These cases trace their genesis to the controversy that has arisen from the forthcoming compulsory retirement of
Chief Justice Puno on May 17, 2010, or seven days after the presidential election. Under Section 4(1), in relation to
Section 9, Article VIII, that “vacancy shall be filled within ninety days from the occurrence thereof” from a “list of at least
three nominees prepared by the Judicial and Bar Council for every vacancy.” Also considering that Section 15, Article VII
(Executive Department) of the Constitution prohibits the President or Acting President from making appointments within
two months immediately before the next presidential elections and up to the end of his term, except temporary
appointments to executive positions when continued vacancies therein will prejudice public service or endanger public
safety.
The JBC, in its en banc meeting of January 18, 2010, unanimously agreed to start the process of filling up the position of
Chief Justice.
Conformably with its existing practice, the JBC “automatically considered” for the position of Chief Justice the five
most senior of the Associate Justices of the Court, namely: Associate Justice Antonio T. Carpio; Associate Justice Renato
C. Corona; Associate Justice Conchita Carpio Morales; Associate Justice Presbitero J. Velasco, Jr.; and Associate
Justice Antonio Eduardo B. Nachura. However, the last two declined their nomination through letters dated January 18,
2010 and January 25, 2010, respectively.
The OSG contends that the incumbent President may appoint the next Chief Justice, because the prohibition
under Section 15, Article VII of the Constitution does not apply to appointments in the Supreme Court. It argues that any
vacancy in the Supreme Court must be filled within 90 days from its occurrence, pursuant to Section 4(1), Article VIII of
the Constitution; that had the framers intended the prohibition to apply to Supreme Court appointments, they could have
easily expressly stated so in the Constitution, which explains why the prohibition found in Article VII (Executive
Department) was not written in Article VIII (Judicial Department); and that the framers also incorporated in Article VIII
ample restrictions or limitations on the President’s power to appoint members of the Supreme Court to ensure its
independence from “political vicissitudes” and its “insulation from political pressures,” such as stringent qualifications for
the positions, the establishment of the JBC, the specified period within which the President shall appoint a Supreme Court
Justice.
A part of the question to be reviewed by the Court is whether the JBC properly initiated the process, there being
an insistence from some of the oppositors-intervenors that the JBC could only do so once the vacancy has occurred (that
is, after May 17, 2010). Another part is, of course, whether the JBC may resume its process until the short list is prepared,
in view of the provision of Section 4(1), Article VIII, which unqualifiedly requires the President to appoint one from the
short list to fill the vacancy in the Supreme Court (be it the Chief Justice or an Associate Justice) within 90 days from the
occurrence of the vacancy.

ISSUE: Whether the incumbent President can appoint the successor of Chief Justice Puno upon his retirement.

HELD: Prohibition under Section 15, Article VII does not apply to appointments to fill a vacancy in the Supreme Court or
to other appointments to the Judiciary.
Two constitutional provisions are seemingly in conflict.
The first, Section 15, Article VII (Executive Department), provides: Section 15. Two months immediately before
the next presidential elections and up to the end of his term, a President or Acting President shall not make appointments,
except temporary appointments to executive positions when continued vacancies therein will prejudice public service or
endanger public safety.
The other, Section 4 (1), Article VIII (Judicial Department), states: Section 4. (1). The Supreme Court shall be
composed of a Chief Justice and fourteen Associate Justices. It may sit en banc or in its discretion, in division of three,
five, or seven Members. Any vacancy shall be filled within ninety days from the occurrence thereof.
Had the framers intended to extend the prohibition contained in Section 15, Article VII to the appointment of
Members of the Supreme Court, they could have explicitly done so. They could not have ignored the meticulous ordering
of the provisions. They would have easily and surely written the prohibition made explicit in Section 15, Article VII as being
equally applicable to the appointment of Members of the Supreme Court in Article VIII itself, most likely in Section 4 (1),
Article VIII. That such specification was not done only reveals that the prohibition against the President or Acting
President making appointments within two months before the next presidential elections and up to the end of the
President’s or Acting President’s term does not refer to the Members of the Supreme Court.
Had the framers intended to extend the prohibition contained in Section 15, Article VII to the appointment of
Members of the Supreme Court, they could have explicitly done so. They could not have ignored the meticulous ordering
of the provisions. They would have easily and surely written the prohibition made explicit in Section 15, Article VII as being
equally applicable to the appointment of Members of the Supreme Court in Article VIII itself, most likely in Section 4 (1),
Article VIII. That such specification was not done only reveals that the prohibition against the President or Acting
President making appointments within two months before the next presidential elections and up to the end of the
President’s or Acting President’s term does not refer to the Members of the Supreme Court.
Section 14, Section 15, and Section 16 are obviously of the same character, in that they affect the power of the
President to appoint. The fact that Section 14 and Section 16 refer only to appointments within the Executive Department
renders conclusive that Section 15 also applies only to the Executive Department. This conclusion is consistent with the
rule that every part of the statute must be interpreted with reference to the context, i.e. that every part must be considered
together with the other parts, and kept subservient to the general intent of the whole enactment. It is absurd to assume
that the framers deliberately situated Section 15 between Section 14 and Section 16, if they intended Section 15 to cover
all kinds of presidential appointments. If that was their intention in respect of appointments to the Judiciary, the framers, if
only to be clear, would have easily and surely inserted a similar prohibition in Article VIII, most likely within Section 4 (1)
thereof.

SECTION 16

12. SARMIENTO VS. MISON


(156 SCRA 154, 1987)

FACTS: The petitioners, who are taxpayers, lawyers, members of the Integrated Bar of the Philippines and professors of
Constitutional Law, seek to enjoin the respondent Salvador Mison from performing the functions of the Office of
Commissioner of the Bureau of Customs and the respondent Guillermo Carague, as Secretary of the Department of
Budget, from effecting disbursements in payment of Mison's salaries and emoluments, on the ground that Mison's
appointment as Commissioner of the Bureau of Customs is unconstitutional by reason of its not having been confirmed by
the Commission on Appointments. The respondents, on the other hand, maintain the constitutionality of respondent
Mison's appointment without the confirmation of the Commission on Appointments.

ISSUES:
1. What are the groups of officers whom the President shall appoint?
2. W/N confirmation of the appointments of Commissioners of the Bureau of Customs by the Commission on
Appointments required.

HELD:
1. Under the provisions of the 1987 Constitution, just quoted, there are four (4) groups of officers whom the
President shall appoint. These four (4) groups, to which we will hereafter refer from time to time, are:
First, the heads of the executive departments, ambassadors, other public ministers and consuls, officers of the armed
forces from the rank of colonel or naval captain, and other officers whose appointments are vested in him in this
Constitution;
Second, all other officers of the Government whose appointments are not otherwise provided for by law;
Third, those whom the President may be authorized by law to appoint;
Fourth, officers lower in rank whose appointments the Congress may by law vest in the President alone.
The first group of officers is clearly appointed with the consent of the Commission on Appointments. Appointments of such
officers are initiated by nomination and, if the nomination is confirmed by the Commission on Appointments, the President
appoints.
Those belonging to second, third and fourth groups may be appointed by the President without such confirmation
with COA.
2. NO. It is evident that the position of Commissioner of the Bureau of Customs (a bureau head) is not one of
those within the first group of appointments where the consent of the Commission on Appointments is required. As a
matter of fact, as already pointed out, while the 1935 Constitution includes "heads of bureaus" among those officers
whose appointments need the consent of the Commission on Appointments, the 1987 Constitution, on the other hand,
deliberately excluded the position of "heads of bureaus" from appointments that need the consent (confirmation) of the
Commission on Appointments.

13. BAUTISTA VS. SALONGA


(172 SCRA 160, 1989)

FACTS: President Aquino designated Mary Bautista as Acting Chairman of the CHR. Later on, the President extended to
Bautista a permanent appointment as Chairman of the Commission. She took her oath of office by virtue of her
appointment as Chairman of the CHR.
Bautista received letters from the COA Secretary requesting her to submit certain information and documents
and to be present at a meeting of the COA Committee on Justice and Judicial and Bar Council and Human Rights, in
connection with her confirmation as Chairman of CHR. However, she refused to submit herself to the COA arguing that
the latter has no jurisdiction to review her appointment as CHR Chairman.
The COA's secretary sent a letter to the executive secretary informing the latter that COA disapproved Bautista's
"ad interim appointment" as Chairman of the CHR, in view of her refusal to submit to the jurisdiction of the COA. It is the
COA's submission that the President decides to the extent another appointment to Bautista, this time, submitting such
appointment/nomination to the COA for confirmation.

ISSUE: Whether or not confirmation of the appointments of the Chairman of the Commission on Human Rights requires
the consent of the COA.

HELD:
No, since the office is not one of those mentioned in the first sentence of Article VII, Section 16, nor is it specified
elsewhere that such appointments needs consent of the Commission, it follows that the appointment by the President of
the Chairman of the CHR is to be made without the review or participation of the Commission on Appointments.
To be more precise, the appointment of the Chairman and Members of the Commission on Human Rights is not
specifically provided for in the Constitution itself, unlike the Chairmen and Members of the Civil Service Commission, the
Commission on Elections and the Commission on Audit, whose appointments are expressly vested by the Constitution in
the President with the consent of the Commission on Appointment.
The President appoints the Chairman and Members of the Commission on Human Rights pursuant to the second
sentence in Section 16, Art. VII, that is, without the confirmation of the Commission on Appointments because they are
among the officers of government "whom he (the President) may be authorized by law to appoint." And Section 2(c),
Executive Order No. 163, 5 May 1987, authorizes the President to appoint the Chairman and Members of the Commission
on Human Rights. It provides:
"(c) The Chairman and the Members of the Commission on Human Rights shall be appointed by the President
for a term of seven years without re-appointment. Appointment to any vacancy shall be only for the unexpired term of the
predecessor."

14. QUINTOS-DELES VS. COMMISSION ON APPOINTMENTS


(177 SCRA 259, 1989)

FACTS: Petitioner and three others were appointed Sectoral Representatives by the President pursuant to Article VII,
Section 16, paragraph 2 and Article XVIII, Section 7 of the Constitution. However, the appointees were not able to take
their oaths and discharge their duties as members of Congress due to the opposition of some congressmen-members of
the Commission on Appointments, who insisted that sectoral representatives must first be confirmed by the respondent
Commission before they could take their oaths and/or assume office as members of the House of Representatives.

ISSUE: W/N appointment of Sectoral Representatives requires confirmation by the Commission on Appointments.

HELD:
YES. Since the seats reserved for sectoral representatives in paragraph 2, Section 5, Art. VI may be filled by
appointment by the President by express provision of Section 7, Art. XVIII of the Constitution, it is undubitable that
sectoral representatives to the House of Representatives are among the "other officers whose appointments are vested in
the President in this Constitution," referred to in the first sentence of Section 16, Art. VII whose appointments are-subject
to confirmation by the Commission on Appointments.
There are appointments vested in the President in the Constitution which, by express mandate of the
Constitution, require no confirmation such as appointments of members of the Supreme Court and judges of lower courts
(Sec. 9, Art. VIII) and the Ombudsman and his deputies (Sec. 9, Art. XI). No such exemption from confirmation had been
extended to appointments of sectoral representatives in the Constitution.

15. CALDERON VS. CARALE


(208 SCRA 254, 1992)

FACTS: Sometime in March 1989, RA 6715 amending the Labor Code was approved. Sec 13 thereof provides that the
Chairman, the Division Presiding Commissioners and other Commissioners shall all be appointed by the President subject
to the confirmation by the Commission on Appointment.
Pursuant to said law, President Aquino appointed the Chairman and Commissioners of the NLRC representing
the public workers and employees sectors. The appointment stated that the appointees may qualify and enter upon the
performance of the duties of the office.
This petition for prohibition questions the constitutionality of the permanent appointment extended by the
President without submitting the same to the Commission on Appointment for confirmation pursuant to RA 6715.
The Solicitor General contends that RA 6715 transgresses Sec 16 Art VII by expanding the confirmation power of the
Commission on Appointments without Constitutional basis.

ISSUES:
1. May the Congress expand the list of those whose appointment needs confirmation by the Commission on
Appointments?
2. When is confirmation by COA can be said to be required and not?

HELD:
1. NO, the Court held that the Congress may not expand the list of appointments needing confirmation.

The NLRC Chairman and Commissioners fall within the second sentence of Section 16, Article VII of the
Constitution, more specifically under the "third groups" of appointees referred to in Mison, i.e. those whom the President
may be authorized by law to appoint. Undeniably, the Chairman and Members of the NLRC are not among the officers
mentioned in the first sentence of Section 16, Article VII whose appointments requires confirmation by the Commission on
Appointments. To the extent that RA 6715 requires confirmation by the Commission on Appointments of the appointments
of respondents Chairman and Members of the National Labor Relations Commission, it is unconstitutional because:
a. it amends by legislation, the first sentence of Sec. 16, Art. VII of the Constitution by adding thereto
appointments requiring confirmation by the Commission on Appointments; and
b. it amends by legislation the second sentence of Sec. 16, Art. VII of the Constitution, by imposing the
confirmation of the Commission on Appointments on appointments which are otherwise entrusted only with the President

2. Confirmation by the Commission on Appointments is required only for presidential appointees mentioned in
the first sentence of Section 16, Article VII, including, those officers whose appointments are expressly vested by the
Constitution itself in the president (like sectoral representatives to Congress and members of the constitutional
commissions of Audit, Civil Service and Election).

Confirmation is not required when the President appoints other government officers whose appointments are not
otherwise provided for by law or those officers whom he may be authorized by law to appoint (like the Chairman and
Members of the Commission on Human Rights). Also, as observed in Mison, when Congress creates inferior offices but
omits to provide for appointment thereto, or provides in an unconstitutional manner for such appointments, the officers are
considered as among those whose appointments are not otherwise provided for by law.
16. TARROSA VS. SINGSON
(232 SCRA 553, 1994)

FACTS: President Ramos appointed respondent Singson as Governor of the Bangko Sentral. Petitioner argues that this
appointment is null and void since it was not submitted for confirmation to the COA. The petition is anchored on the
provisions of Section 6 of R.A. No. 7653, which established the Bangko Sentral as the Central Monetary Authority of the
Philippines. Section 6, Article II of R.A. No. 7653 provides:
"Sec. 6. Composition of the Monetary Board. The powers and functions of the Bangko Sentral shall be exercised by the
Bangko Sentral Monetary Board, hereafter referred to as the Monetary Board, composed of seven (7) members appointed
by the President of the Philippines for a term of six (6) years.
The seven (7) members are:
(a) The Governor of the Bangko Sentral, who shall be the Chairman of the Monetary Board. The Governor of the
Bangko Sentral shall be head of a department and his appointment shall be subject to confirmation by the Commission on
Appointments. Whenever the Governor is unable to attend a meeting of the Board, he shall designate a Deputy Governor
to act as his alternate: Provided, That in such event, the Monetary Board shall designate one of its members as acting
Chairman . . ." (Underlining supplied).
In their comment, respondents claim that Congress exceeded its legislative powers in requiring the confirmation
by the Commission on Appointments of the appointment of the Governor of the Bangko Sentral. They contend that an
appointment to the said position is not among the appointments which have to be confirmed by the Commission on
Appointments, citing Section 16 of Article VII of the Constitution.

ISSUE: May the Congress expand the confirmation powers of the Commission on Appointments and require appointment
of other government officials not expressly mentioned in the first sentence of Sec. 16 of Article 7 of the Constitution?

HELD: NO. In this case the Court used the same ruling as what was held in the case of Calderon vs. Corale, where the
ruled was that Congress cannot by law expand the confirmation powers of the COA and require confirmation of
appointments of other government officials not expressly mentioned in the first sentence of Section 16 of Article VII of the
Constitution.

17. FLORES VS. DRILON


(223 SCRA 568, 1993)

FACTS: Mayor Gordon of Olongapo City was appointed Chairman of SBMA on account of RA 7227. Under said law, for
the first year of its effectiveness, the mayor of Olongapo shall be appointed as chairman of the SBMA.
It was argued that said provision violates Sec. 7 Art IX:-B which provides: that no elective official shall be eligible
for appointment in any capacity to any public officer or position during his tenure.
It was contended that the prohibition cannot be applied due to the presence of a law authorizing the appointment.
The Local Government Code permits the appointment of local elective official to another position or post.

ISSUE: Whether the proviso in Sec. 13, par. (d), of R.A. 7227 which states, "Provided, however, That for the first year of
its operations from the effectivity of this Act, the mayor of the City of Olongapo shall be appointed as the chairman and
chief executive officer of the Subic Authority," violates the constitutional proscription against appointment or designation of
elective officials to other government posts.

HELD:
YES. In the case before us, the subject proviso directs the President to appoint an elective official, i.e., the Mayor
of Olongapo City, to other government posts (as Chairman of the Board and Chief Executive Officer of SBMA). Since this
is precisely what the constitutional proscription seeks to prevent, it needs no stretching of the imagination to conclude that
the proviso contravenes Sec. 7, first part., Art. IX-B, of the Constitution. Here, the fact that the expertise of an elective
official may be most beneficial to the higher interest of the body politic is of no moment.
It is argued that Sec. 94 of the Local Government Code (LGC) permits the appointment of a local elective official
to another post if so allowed by law or by the primary functions of his office. 8 But, the contention is fallacious. Section 94
of the LGC is not determinative of the constitutionality of Sec. 13, par. (d), of R.A. 7227, for no legislative act can prevail
over the fundamental law of the land.
18. LUEGO VS. CIVIL SERVICE COMMISSION
(143 SCRA 327, 1986)

FACTS: The petitioner was appointed Administrative Officer II by the city mayor, Mayor Solon. The appointment was
described as ―permanent‖ but the Civil Service Service Commission approved it as ―temporary‖ subject to the outcome
of the protest of the respondent.
The Civil Service Commission decided that respondent was better qualified, revoked the appointment of
petitioner and ordered the appointment of respondent in his place. The private responded was so appointed by the new
mayor, Mayor Duterte.
The petitioner, invoking his earlier ―permanent‖ appointment, is now before the Court to question that order and
the private respondent‘s title.

ISSUES:
1. Is the Civil Service Commission authorized to disapprove a permanent appointment on the ground that another person
is better qualified than the appointee and, on the basis of this finding, order his replacement by the latter?
2. W/N the Civil Service Commission has the power to make a permanent appointment into a temporary one.

HELD:
1. NO. The Civil Service Commission is without authority to revoke an appointment because of its belief that
another person was better qualified, which is an encroachment on the discretion vested solely in the city mayor.
2. NO. While the principle is correct, and we have applied it many times, it is not correctly applied in this case.
The argument begs the question. The appointment of the petitioner was not temporary but permanent and was therefore
protected by Constitution. The appointing authority indicated that it was permanent, as he had the right to do so, and it
was not for the respondent Civil Service Commission to reverse him and call it temporary.

19. POBRE VS. MENDIETA


(224 SCRA 738, 1993)

FACTS: This controversy began when the term of office of Honorable Francia as PRC Commissioner/Chairman expired.
At that time, Mendieta was the senior associate Commissioner and Pobre was the second associate Commissioner of the
PRC.
Then the executive secretary sought the opinion of acting secretary of justice on whether the President may
appoint as Commissioner/Chairman of the PRC any person other than the Senior Associate Commissioner. Acting
secretary of justice answered that Sec. 2 of PD 223 does not limit or restrict the appointing power of the President.
President Aquino then appointed the petitioner, then an Associate Commissioner, as the PRC
Commissioner/Chairman.
Mendieta filed a petition for declaratory relief contesting Pobre‘s appointment as Chairman of the PRC because
he allegedly succeeded Francia as PRC Chairman by operation of law. The trial court ruled in favor of Mendieta. Hence,
this petition.

ISSUE: W/N the vacancy in the commission shall be filled by ―succession‖ or ―operation of law.

HELD: NO. The Court finds unacceptable the view that every vacancy in the Commission (except the position of "junior"
Associate Commissioner) shall be filled by "succession" or by "operation of law" for that would deprive the President of his
power to appoint a new PRC Commissioner and Associate Commissioners -- "all to be appointed by the President" under
P.D. No. 223. The absurd result would be that the only occasion for the President to exercise his appointing power would
be when the position of junior (or second) Associate Commissioner becomes vacant. We may not presume that when the
President issued P.D. No. 223, he deliberately clipped his prerogative to choose and appoint the head of the PRC and
limited himself to the selection and appointment of only the associate commissioner occupying the lowest rung of the
ladder in that agency.

20. ARTURO DE CASTRO V JBC AND GLORIA ARROYO


SECTION 17
21. DRILON VS. LIM
(235 SCRA 135, 1994)

FACTS: Pursuant to section 187 of the Local Government Code (Procedure for approval and effectivity of Tax
Ordianance and Revenue Measures), the Secretary of Justice had, on appeal to him of four oil companies and taxpayer,
declared Manila Revenue Code null and void for non-compliance with the prescribed procedure in the enactment of tax
ordianance (there were no written notices of public hearings nor were copies of the proposed ordinance published).
In a petition for certiorari, RTC revoked Secretary‘s resolution and sustained ordinance holding that all the
procedural requirements had been observed in the enactment of the Manila Revenue Code and that the City of Manila
had not been able to prove such compliance before the Secretary only because he had given it only five days within which
to gather and present to him all the evidence later submitted to the trial court. More importantly, it declared Section 187 of
the LGC as unconstitutional insofar as it empowered the Secretary of Justice to review tax ordinance and inferentially to
annul them. His conclusion was that the challenged section gave the Secretary the power of control and not of supervision
only. The 1987 Constitution provides that President shall exercise general supervision over local governments.

ISSUES:
1. Distinguish control from supervision.
2. W/N Section 187 of the LGC gave the Secretary the power of control and not supervision only.

HELD:
1. An officer in control lays down the rules in the doing of an act. It they are not followed, he may, in his discretion, order
the act undone or re-done by his subordinate or he may even decide to do it himself. Supervision does not cover such
authority. The supervisor or superintendent merely sees to it that the rules are followed, but he himself does not lay down
such rules, nor does he have the discretion to modify or replace them. If the rules are not observed, he may order the
work done or re-done but only to conform to the prescribed rules. He may not prescribe his own manner for the doing of
the act. He has no judgment on this matter except to see to it that the rules are followed.

2. NO. In the opinion of the Court, Secretary Drilon did precisely this (See No.1), and no more nor less than this, and so
performed an act not of control but of mere supervision.

Secretary Drilon did set aside the Manila Revenue Code, but he did not replace it with his own version of what
the Code should be. He did not pronounce the ordinance unwise or unreasonable as a basis for its annulment. He did not
say that in his judgment it was a bad law. What he found only was that it was illegal. All he did in reviewing the said
measure was determine if the petitioners were performing their functions is accordance with law, that is, with the
prescribed procedure for the enactment of tax ordinances and the grant of powers to the city government under the Local
Government Code. As we see it, that was an act not of control but of mere supervision
WHEREFORE, the judgment is hereby rendered REVERSING the challenged decision of the Regional Trial
Court insofar as it declared Section 187 of the Local Government Code unconstitutional but AFFIRMING its finding that
the procedural requirements in the enactment of the Manila Revenue Code have been observed.

22. VILLENA VS. SECRETARY OF INTERIOR


(67 PHIL 451)

FACTS: It appears that the Division of Investigation of the Department of Justice, upon the request of the Secretary of the
Interior, conducted an inquiry into the conduct of the petitioner, as a result of which the latter was found to have committed
bribery, extortion, malicious abuse of authority and unauthorized practice of the law profession. The respondent
recommended to the President of the Philippines the suspension of the petitioner to prevent possible coercion of
witnesses, which recommendation was granted. The Secretary of the Interior suspended the petitioner from office, and
then and thereafter wired the Provincial Governor of Rizal with instruction that the petitioner be advised accordingly. The
respondent wrote the petitioner a letter, specifying the many charges against him and notifying him of the designation of a
special investigator to investigate the charges. The special investigator forthwith notified the petitioner that the formal
investigation would be commenced on March 28, 1939. Hence, the petition for preliminary injunction against the Secretary
of Interior to restrain him and his agents from preceding with the investigation of petitioner which was scheduled to take
place on March 28, 1939.

ISSUES:
1. W/N the Secretary of Interior has the power to order an investigation.
2. W/N the Secretary of Interior has the power to suspend.

HELD:
1. YES. Supervision is not a meaningless thing. It is an active power. It is certainly not without limitation, but it at
least implies authority to inquire into facts and conditions in order to render the power real and effective. If supervision is
to be conscientious and rational, and not automatic and brutal, it must be founded upon knowledge of actual facts and
conditions disclosed after careful study and investigation. The principle there enunciated is applicable with equal force to
the present case.

The Secretary of the Interior is invested with authority to order the investigation of the charges against the
petitioner and to appoint a special investigator for that purpose.

2. YES. The Secretary of Interior is empowered to investigate the charges against the pwtitioner and to appoint a
special investigator for that purpose, preventive suspension may be a means by which to carry into effect a fair and
impartial investigation.

23. LACSON-MAGALLANES CO., INC. VS. PANO


(21 SCRA 395, 1967)

FACTS: Jose Magallanes, a permittee and actual occupant of a 1,103-hectare pasture land, ceded his rights and interests
to a portion thereof to plaintiff. Subsequently, the portion Magallanes ceded to plaintiff was officially released from the
forest zone as pasture land and declared agricultural land.
Jose Paño and nineteen other claimants applied for the purchase of 90 hectares of the released area. Plaintiff corporation
in turn filed its own sales application covering the entire released area. This was protested by Paño and his companions,
claiming that they are actual occupants of the part thereof covered by their own sales application.
The Director of Lands rendered judgment, giving due course to the application of plaintiff corporation, and
dismissing the claim of Jose Paño and his companions. A move to reconsider failed. An appeal was made but the
Secretary of Agriculture and Natural Resources dismissed the same.
When the case was elevated to the President of the Philippines, Executive Secretary Juan Pajo, by authority of the
President, modified the decision of the Director of Lands as affirmed by the Secretary of Agriculture and Natural
Resources.
Plaintiff corporation took this decision to the trial court, praying that judgment be rendered declaring that the
decision of the Secretary of Agriculture and Natural Resources has full force and effect. Plaintiff's mainstay is Section 4 of
Commonwealth Act 141. The precept there is that decisions of the Director of Lands "as to questions of fact shall be
conclusive when approved" by the Secretary of Agriculture and Natural Resources. Plaintiff's trenchant claim is that this
statute is controlling not only upon courts but also upon the President.

ISSUE: May the President through his executive secretary undo an act of the Director of Lands, which a law provides that
such act will be conclusive when affirmed by the Secretary of Agriculture and Natural Resources OR W/N the
administrative decision could still be appealed to the President?

HELD:
YES. The President's duty to execute the law is of constitutional origin. So, too, is his control of all executive
departments. Thus it is, that department heads are men of his confidence. His is the power to appoint them; his, too, is the
privilege to dismiss them at pleasure. Naturally, he controls and directs their acts. Implicit then is his authority to go over,
confirm, modify or reverse the action taken by his department secretaries. In this context, it may not be said that the
President cannot rule on the correctness of a decision of a department secretary.
It may be stated that the right to appeal to the President reposes upon the President's power of control over the
executive departments. And control simply means the power of an officer to alter or modify or nullify or set aside what a
subordinate officer had done in the performance of his duties and to substitute the judgment of the former for that of the
latter.

24. CITY OF ILIGAN V. DIRECTOR OF LANDS


(158 SCRA 158 [1988])

FACTS: Proclamation No. 335 was issued, withdrawing from sale or settlement and reserved for the use of the NPC
certain parcels of the public domain.
Meanwhile, the NPC constructed Maria Cristina Fertilizer Plant, which was sold, ceded, transferred and conveyed to
Marcelo Tire and Rubber Corporation, including the right of occupancy and use of the land described in Proclamation 335,
Series of 1952.
Proclamation No. 20, Series of 1962, and Proclamation 198, Series of 1964, were subsequently issued,
excluding from the operation of Proclamation No. 335, Series of 1952, certain areas occupied by the Maria Cristina
Fertilizer Plant, and declaring the same open to disposition under the provisions of Public Land.
The Marcelo Steel Corporation and/or the Maria Cristina Fertilizer Plant, through the President, Jose P. Marcelo
filed in the Bureau of Lands a Miscellaneous Sales Application for tracts of lands for industrial purposes. Director of Lands
advised the public that the Bureau of Lands will sell to the highest qualified bidder the tract of land covered by
Miscellaneous Sales application.
Proclamation No. 469 was later issued, which exclude from the reservation made in favor of the NPC, existing
under Proclamation No. 335, Series of 1952, and Proclamation No. 20, Series of 1962, certain parcels of land embraced
therein." Lots 1,1-a, 3 and 4, containing approximately an area of 29,681 square meters are described therein. The
Proclamation further stated "that upon the recommendation of the Secretary of Agriculture and Natural Resources and
pursuant to Section 60 of C.A. No. 141, I do hereby grant, donate and transfer the aforementioned parcels of land
including the foreshores thereof, in favor of Iligan City."
The Mayor of Iligan City wrote the Director of Lands to inform him that the City of Iligan is the owner in fee simple
of Lots 1, 1-a, 3 and 4 including the foreshores thereof by virtue of Proclamation No. 469, Series of 1965, and requesting
that the said property be excluded from the proposed auction sale." No action was taken on this request for exclusion.
Hence, the City of Iligan filed a complaint for injunction with preliminary injunction against the Director of Lands,
District Land Officer of Lanao del Norte and the Marcelo Steel Corporation to enjoin and stop the sale and/or disposition of
the afore described parcels of land.
President Marcos issued Proclamation No. 94 excluding from the operation of Proclamation No. 469 certain portions of
the land embraced therein, situated in Iligan City and declaring the same open to disposition. Said portions of land, as
described therein are Lots 1-a, 2-a and 3 of the parcels of land in question.
After the trial on the merits, the court dismissed the complaint and dissolved the writ of preliminary injunction. On
appeal, the records of the case were certified to this Court as the issue of the validity of any executive order and the errors
or the questions of the law raised are within the exclusive jurisdiction of this Court.

ISSUE: Whether or not the President has the power to grant portions of public domain to any government entity like the
city of Iligan.

HELD:
YES. the Secretary of Agriculture and Natural Resources is the executive officer-in-charged with the duty of
carrying out the provision of the Public Land Act thru the Director of Lands who acts under his immediate control.
Section 4 thereof, also provides:
"Sec. 4. Subject to said control, the Director of Lands shall have direct executive control of the survey,
classification, lease, sale or any other form of concession or disposition and management of the lands of the public
domain, and his decisions as to questions of fact shall be conclusive when approved by the Secretary of Agriculture and
Natural Resources."
Since it is the Director of Lands who has direct executive control among others in the lease, sale or any form of
concession or disposition of the land of the public domain subject to the immediate control of the Secretary of Agriculture
and Natural Resources, and considering that under the Constitution the President of the Philippines has control over all
executive departments, bureaus, and offices, etc., 15 the President of the Philippines has therefore the same authority to
dispose of portions of the public domain as his subordinates, the Director of Lands, and his alter ego the Secretary of
Agriculture and Natural Resources.
25. GASCON VS. ARROYO
(178 SCRA 582, 1989)

FACTS: The Lopez family is the owner of two television stations, namely: Channels 2 and 4 which they have operated
through the ABS-CBN Broadcasting Corporation.
When martial law was declared, TV Channel 4 was closed by the military; thereafter, its facilities were taken over
by the Kanlaon Broadcasting System which operated it as a commercial TV station.
In 1978, the said TV station and its facilities were taken over by the National Media Production Center (NMPC),
which operated it as the Maharlika Broadcasting System TV 4 (MBS-4).
After the 1986 EDSA revolution, the PCGG sequestered the aforementioned TV Stations, and, thereafter, the Office of
Media Affairs took over the operation of TV Channel 4.
The Lopez family, through counsel, requested President Aquino to order the return to the Lopez family of TV
Stations 2 and 4. They made a written request to the PCGG for the return of TV Station Channel 2. The PCGG approved
the return of TV Station Channel 2 to the Lopez family. The return was made on 18 October 1986.
Thereafter, the Lopez family requested for the return of TV Station Channel 4. Acting upon the request,
respondent Executive Secretary, by authority of the President, entered into with the ABS-CBN Broadcasting Corporation,
represented by its President, Eugenio Lopez, Jr., an "Agreement to Arbitrate", 3 pursuant to which an Arbitration
Committee was created, composed of Atty. Catalino Macaraig, Jr., for the Republic of the Philippines, Atty. Pastor del
Rosario, for ABS-CBN, and retired Justice Vicente Abad Santos, as Chairman.
Thereupon, petitioners, as taxpayers, filed the instant petition.

ISSUE: W/N the Executive Secretary has the power and authority to enter into an Agreement to Arbitrate.

HELD:
YES. Respondent Executive Secretary has the power and authority to enter into the Agreement to arbitrate with
the ABS-CBN Broadcasting Corporation as he acted for and in behalf of the President when he signed it.
Under the Provisional Constitution of the Republic of the Philippines (also known as the Freedom Constitution),
which was in force and effect when the "Agreement to Arbitrate" was signed by the parties thereto on 6 January 1987, the
President exercised both the legislative and executive powers of the Government. As Chief Executive, the President was
(and even now) "assisted by a Cabinet" composed of Ministers (now Secretaries), who were appointed by and
accountable to the President. In other words, the Members of the cabinet, as heads of the various departments, are the
assistants and agents of the Chief Executive, and, except in cases where the Chief Executive is required by the
Constitution or the law to act in person, or where the exigencies of the situation demand that he act personally, the
multifarious executive and administrative functions of the Chief Executive are performed by and through the executive
departments, and the acts of the heads of such departments, performed in the regular course of business, are, unless
disapproved or reprobated by the Chief Executive, presumptively the acts of the Chief Executive.
Respondent Executive Secretary had, therefore, the power and authority to enter into the "Agreement to Arbitrate" with
the ABS-CBN Broadcasting Corporation, as he acted for and in behalf of the President when he signed it; hence, the
aforesaid agreement is valid and binding upon the Republic of the Philippines, as a party thereto.

26. KILUSANG BAYAN VS. DOMINGUEZ


(205 SCRA 92, 1992)

FACTS: The Municipal Government of Muntinlupa entered into a contract with the Kilusang Bayan sa Paglilingkod ng
mga Nagtitinda sa Bagong Pamilihang Bayan ng Muntinlupa (kilusan) for the latter's management and operation of the
Muntinlupa Public Market. When Ignacio Bunye (petitioner in GR 91927) became Mayor of Muntinlupa, he directed a
review of such contract, claiming that the virtual, 50-year term agreement was contrary to Sec. 143 (3) of BP 337. He
sought opinions from the COA and the Metro Manila Commission after which the latter granted the Municipality the
authority to take the necessary legal steps for the cancellation of the above contract.
Consequently, upon the presentation made by Bunye with the Municipal Council, the latter approved Resolution
No. 45 abrogating the contract. Bunye, together with men from the PC, proceeded to the public market and announced to
the general public that the Municipality was taking over the management and operation of the facility therein. The officers
of the Kilusan filed suit for breach of contract and damages, and continued holding office in the KB Building under their
respective official capacities.
Bunye, together with some heavily armed men, forcibly opened the doors of the offices of petitioners purportedly to serve
them the Order of then Sec. of Agriculture Carlos Dominguez, ordering 1) the take over by the Department of Agriculture
of the management over the public market pursuant to the Department Regulatory and Supervisory Power under Sec. 8 of
PD 175 and Sec. 4 of EO No.3; 2) the creation of a Management Committee which shall assume the management of
Kilusan; and 3) the disband, of the Board of Directors and 4) the turn over of all assets, properties and records to the
Management Committee. Petitioners filed this petition praying that the Order to be declared null and void as the
respondent Secretary acted without or in excess of jurisdiction in issuing the order.

ISSUE: W/N 28 October 1988 Order of respondent Secretary of Agriculture is without or in excess of jurisdiction?

HELD:
YES. P.D. No. 175 and the by-laws of the KBMBPM explicitly mandate the manner by which directors and
officers are to be removed. The Secretary should have known better than to disregard these procedures and rely on a
mere petition by the general membership of the KBMBPM and an on-going audit by Department of Agriculture auditors in
exercising a power which he does not have, expressly or impliedly.
An administrative officer has only such powers as are expressly granted to him and those necessarily implied in the
exercise thereof. These powers should not be extended by implication beyond what may be necessary for their just and
reasonable execution.
Supervision and control include only the authority to: (a) act directly whenever a specific function is entrusted by
law or regulation to a subordinate; (b) direct the performance of duty; restrain the commission of acts; (c) review, approve,
reverse or modify acts and decisions of subordinate officials or units; (d) determine priorities in the execution of plans and
programs; and (e) prescribe standards, guidelines, plans and programs.
Specifically, administrative supervision is limited to the authority of the department or its equivalent to: (1)
generally oversee the operations of such agencies and insure that they are managed effectively, efficiently and
economically but without interference with day-to-day activities; (2) require the submission of reports and cause the
conduct of management audit, performance evaluation and inspection to determine compliance with policies, standards
and guidelines of the department; (3) take such action as may be necessary for the proper performance of official
functions, including rectification of violations, abuses and other forms of mal-administration; (4) review and pass upon
budget proposals of such agencies but may not increase or add to them.

SECTION 18
27. IBP vs. Zamora
G.R. No.141284, August 15, 2000

Facts: Invoking his powers as Commander-in-Chief under Sec. 18, Art. VII of the Constitution, the President directed the
AFP Chief of Staff and PNP Chief to coordinate with each other for the proper deployment and utilization of the Marines to
assist the PNP in preventing or suppressing criminal or lawless violence. The President declared that the services of the
Marines in the anti-crime campaign are merely temporary in nature and for a reasonable period only, until such time when
the situation shall have improved. The IBP filed a petition seeking to declare the deployment of the Philippine Marines null
and void and unconstitutional.

Issues:
(1) Whether or not the President’s factual determination of the necessity of calling the armed forces is subject to judicial
review
(2) Whether or not the calling of the armed forces to assist the PNP in joint visibility patrols violates the constitutional
provisions on civilian supremacy over the military and the civilian character of the PNP

Held: When the President calls the armed forces to prevent or suppress lawless violence, invasion or rebellion, he
necessarily exercises a discretionary power solely vested in his wisdom. Under Sec. 18, Art. VII of the Constitution,
Congress may revoke such proclamation of martial law or suspension of the privilege of the writ of habeas corpus and the
Court may review the sufficiency of the factual basis thereof. However, there is no such equivalent provision dealing with
the revocation or review of the President’s action to call out the armed forces. The distinction places the calling out power
in a different category from the power to declare martial law and power to suspend the privilege of the writ of habeas
corpus, otherwise, the framers of the Constitution would have simply lumped together the 3 powers and provided for their
revocation and review without any qualification.
The reason for the difference in the treatment of the said powers highlights the intent to grant the President the widest
leeway and broadest discretion in using the power to call out because it is considered as the lesser and more benign
power compared to the power to suspend the privilege of the writ of habeas corpus and the power to impose martial law,
both of which involve the curtailment and suppression of certain basic civil rights and individual freedoms, and thus
necessitating safeguards by Congress and review by the Court.
In view of the constitutional intent to give the President full discretionary power to determine the necessity of calling
out the armed forces, it is incumbent upon the petitioner to show that the President’s decision is totally bereft of factual
basis. The present petition fails to discharge such heavy burden, as there is no evidence to support the assertion that
there exists no justification for calling out the armed forces.
The Court disagrees to the contention that by the deployment of the Marines, the civilian task of law enforcement is
“militarized” in violation of Sec. 3, Art. II of the Constitution. The deployment of the Marines does not constitute a breach of
the civilian supremacy clause. The calling of the Marines constitutes permissible use of military assets for civilian law
enforcement. The local police forces are the ones in charge of the visibility patrols at all times, the real authority belonging
to the PNP
Moreover, the deployment of the Marines to assist the PNP does not unmake the civilian character of the police force.
The real authority in the operations is lodged with the head of a civilian institution, the PNP, and not with the military.
Since none of the Marines was incorporated or enlisted as members of the PNP, there can be no appointment to civilian
position to speak of. Hence, the deployment of the Marines in the joint visibility patrols does not destroy the civilian
character of the PNP.

28. OLAGUER VS. MILITARY COMMISSION NO. 34


(150 SCRA 144, 1987)

FACTS: The petitioners were charged for subversion.


The respondent Chief of Staff of the AFP created the respondent Military Commission No. 34 to try the criminal
case filed against the petitioners. An amended charge sheet was filed for seven offenses, namely: (1) unlawful possession
of explosives and incendiary devices; (2) conspiracy to assassinate President and Mrs. Marcos; (3) conspiracy to
assassinate cabinet members Juan Ponce Enrile, Francisco Tatad and Vicente Paterno; (4) conspiracy to assassinate
Messrs. Arturo Tangco, Jose Roño and Onofre Corpus; (5) arson of nine buildings; (6) attempted murder of Messrs.
Leonardo Perez, Teodoro Valencia and Generals Romeo Espino and Fabian Ver; and (7) conspiracy and proposal to
commit rebellion, and inciting to rebellion. Sometime thereafter, trial ensued.
In the course of the proceedings the petitioners went to this Court and filed the instant Petition for prohibition and
habeas corpus. They sought to enjoin the respondent Military Commission No. 34 from proceeding with the trial of their
case. They likewise sought their release from detention by way of a writ of habeas corpus. The thrust of their arguments is
that military commissions have no jurisdiction to try civilians for offenses alleged to have been committed during the
period of martial law.

ISSUE: W/N military commissions have no jurisdiction to try civilians for offenses alleged to have been committed during
the period of martial law.

HELD:
Military commission has no jurisdiction to try civilians when the civil courts are open.
Due process of law demands that in all criminal prosecutions (where the accused stands to lose either his life or
his liberty), the accused shall be entitled to, among others, a trial. The trial contemplated by the due process clause of the
Constitution, in relation to the Charter as a whole, is a trial by judicial process, not by executive or military process. Military
commissions or tribunals, by whatever name they are called, are not courts within the Philippine judicial system.
29. SANLAKAS VS. EXECUTIVE SECRETARY
[421 SCRA 656; G.R. No. 159085; 3 Feb 2004]

Facts: During the wee hours of July 27, 2003, some three-hundred junior officers and enlisted men of the AFP, acting
upon instigation, command and direction of known and unknown leaders have seized the Oakwood Building in Makati.
Publicly, they complained of the corruption in the AFP and declared their withdrawal of support for the government,
demanding the resignation of the President, Secretary of Defense and the PNP Chief. These acts constitute a violation of
Article 134 of the Revised Penal Code, and by virtue of Proclamation No. 427 and General Order No. 4, the Philippines
was declared under the State of Rebellion. Negotiations took place and the officers went back to their barracks in the
evening of the same day. On August 1, 2003, both the Proclamation and General Orders were lifted, and Proclamation
No. 435, declaring the Cessation of the State of Rebellion was issued.
In the interim, however, the following petitions were filed: (1) SANLAKAS AND PARTIDO NG MANGGAGAWA
VS. EXECUTIVE SECRETARY, petitioners contending that Sec. 18 Article VII of the Constitution does not require the
declaration of a state of rebellion to call out the AFP, and that there is no factual basis for such proclamation. (2)SJS
Officers/Members v. Hon. Executive Secretary, et al, petitioners contending that the proclamation is a circumvention of the
report requirement under the same Section 18, Article VII, commanding the President to submit a report to Congress
within 48 hours from the proclamation of martial law. Finally, they contend that the presidential issuances cannot be
construed as an exercise of emergency powers as Congress has not delegated any such power to the President. (3) Rep.
Suplico et al. v. President Macapagal-Arroyo and Executive Secretary Romulo, petitioners contending that there was
usurpation of the power of Congress granted by Section 23 (2), Article VI of the Constitution. (4) Pimentel v. Romulo, et al,
petitioner fears that the declaration of a state of rebellion "opens the door to the unconstitutional implementation of
warrantless arrests" for the crime of rebellion.

Issues:
(1) Whether or Not Proclamation No. 427 and General Order No. 4 are constitutional?
(2) Whether or Not the petitioners have a legal standing or locus standi to bring suit?

Held: The Court rendered that the both the Proclamation No. 427 and General Order No. 4 are constitutional. Section 18,
Article VII does not expressly prohibit declaring state or rebellion. The President in addition to its Commander-in-Chief
Powers is conferred by the Constitution executive powers. It is not disputed that the President has full discretionary power
to call out the armed forces and to determine the necessity for the exercise of such power. While the Court may examine
whether the power was exercised within constitutional limits or in a manner constituting grave abuse of discretion, none of
the petitioners here have, by way of proof, supported their assertion that the President acted without factual basis. The
issue of the circumvention of the report is of no merit as there was no indication that military tribunals have replaced civil
courts or that military authorities have taken over the functions of Civil Courts.
The issue of usurpation of the legislative power of the Congress is of no moment since the President, in declaring
a state of rebellion and in calling out the armed forces, was merely exercising a wedding of her Chief Executive and
Commander-in-Chief powers. These are purely executive powers, vested on the President by Sections 1 and 18, Article
VII, as opposed to thedelegated legislative powers contemplated by Section 23 (2), Article VI. The fear on warrantless
arrest is unreasonable, since any person may be subject to this whether there is rebellion or not as this is a crime
punishable under the Revised Penal Code, and as long as a valid warrantless arrest is present.
Legal standing or locus standi has been defined as a personal and substantial interest in the case such that the
party has sustained or will sustain direct injury as a result of the governmental act that is being challenged. The gist of the
question of standing is whether a party alleges "such personal stake in the outcome of the controversy as to assure that
concrete adverseness which sharpens the presentation of Issue upon which the court depends for illumination of difficult
constitutional questions. Based on the foregoing, petitioners Sanlakas and PM, and SJS Officers/Members have no legal
standing to sue. Only petitioners Rep. Suplico et al. and Sen. Pimentel, as Members of Congress, have standing to
challenge the subject issuances. It sustained its decision in Philippine Constitution Association v. Enriquez, that the extent
the powers of Congress are impaired, so is the power of each member thereof, since his office confers a right to
participate in the exercise of the powers of that institution.
30. Gudani vs. Senga
G.R. No. 170165, August 15, 2006

FACTS: Petitioners Gen. Gudani and Lieutenant Colonel Balutan are high-ranking officers of Philippine Marines assigned
to the Philippine Military Academy (PMA) in Baguio City. Senator Biazon invited several senior officers of the military to
appear at a public hearing before a Senate Committee to clarify allegations of massive cheating and the surfacing of
copies of an audio excerpt purportedly of a phone conversation between the President and then Commission on Elections
Commissioner Garcillano. At the time of the 2004 elections, Gen. Gudani had been designated as commander, and Col.
Balutan a member, of “Joint Task Force Ranao” by the AFP Southern Command. Armed Forces of the Philippines (AFP)
Chief of Staff Lt . Gen. Senga were among the several AFP officers also received a letter invitation from Sen. Biazon to
attend the hearing. But only Gen. Gudani, and Col. Balutan attended the invitation from Sen. Biazon.

Thereafter, the Office of the Chief of Staff of the AFP issued a Memorandum addressed to Gen. Baloing. It was
signed by Lt. Col. Hernando DCA Iriberri in behalf of Gen. Senga. Noting that Gen. Gudani and Col. Balutan had been
invited to attend the Senate Committee hearing, the Memorandum directed the two officers to attend the hearing.
Conformably, Gen. Gudani and Col. Balutan filed their respective requests for travel authority addressed to the PMA
Superintendent.
However, Gen. Senga did not attend to the requested hearing as per instruction from the President that NO AFP
PERSONNEL SHALL APPEAR BEFORE ANY CONGRESSIONAL OR SENATE HEARING WITHOUT HER APPROVAL.
`
While Gen. Gudani and Col. Balutan had concluded their testimony, the office of Gen. Senga issued a statement
which noted that the two had appeared before the Senate Committee “in spite of the fact that a guidance has been given
that a Presidential approval should be sought prior to such an appearance;” that such directive was “in keeping with the
time[-]honored principle of the Chain of Command;” and that the two officers “disobeyed a legal order, in violation of
A[rticles of] W[ar] 65 (Willfully Disobeying Superior Officer), hence they will be subjected to General Court Martial
proceedings x x x” Both Gen. Gudani and Col. Balutan were likewise relieved of their assignments then.
On the very day of the hearing, the President issued Executive Order (E.O.) 464. The Office of the Solicitor
General notes that the E.O. “enjoined officials of the executive department including the military establishment from
appearing in any legislative inquiry without her approval.
Now, petitioners seek the annulment of a directive from the President enjoining them and other military officers
from testifying before Congress without the President’s consent. Petitioners also pray for injunctive relief against a
pending preliminary investigation against them, in preparation for possible court-martial proceedings, initiated within the
military justice system in connection with petitioners’ violation of the aforementioned directive.
The Court has to resolve whether petitioners may be subjected to military discipline on account of their defiance
of a direct order of the AFP Chief of Staff.

ISSUE: Whether or not E.O. 464 which provides among others that NO AFP PERSONNEL SHALL APPEAR BEFORE
ANY CONGRESSIONAL OR SENATE HEARING WITHOUT HER APPROVAL is unconstitutional?

RULING: The Petition is dismissed.


Is EO 464 constitutional or not, or may the President prevent a member of the armed forces from testifying before a
legislative inquiry?
Insofar as E.O. 464 compelled officials of the executive branch to seek prior presidential approval before
appearing before Congress, the notion of executive control also comes into consideration. The impression is wrong. The
ability of the President to require a military official to secure prior consent before appearing in Congress pertains to wholly
different and independent specie of presidential authority—the commander-in-chief powers of the President. By tradition
and jurisprudence, the commander-in-chief powers of the President are not encumbered by the same degree of restriction
as that which may attach to executive privilege or executive control.
We hold that the President has constitutional authority to do so, by virtue of her power as commander-in-chief,
and that as a consequence a military officer who defies such injunction is liable under military justice. At the same time,
we also hold that any chamber of Congress which seeks to appear before it a military officer against the consent of the
President has adequate remedies under law to compel such attendance. Any military official whom Congress summons to
testify before it may be compelled to do so by the President. If the President is not so inclined, the President may be
commanded by judicial order to compel the attendance of the military officer. Final judicial orders have the force of the law
of the land which the President has the duty to faithfully execute.
Again, let it be emphasized that the ability of the President to prevent military officers from testifying before
Congress does not turn on executive privilege, but on the Chief Executive’s power as commander-in-chief to control the
actions and speech of members of the armed forces. The President’s prerogatives as commander-in-chief are not
hampered by the same limitations as in executive privilege. The commander-in-chief provision in the Constitution is
denominated as Section 18, Article VII, which begins with the simple declaration that “[t]he President shall be the
Commander-in-Chief of all armed forces of the Philippines x x x Outside explicit constitutional limitations, such as those
found in Section 5, Article XVI, the commander-in-chief clause vests on the President, as commander-in-chief, absolute
authority over the persons and actions of the members of the armed forces. Such authority includes the ability of the
President to restrict the travel, movement and speech of military officers, activities which may otherwise be sanctioned
under civilian law.
Reference to Kapunan, Jr. v. De Villa is useful in this regard. Lt. Col. Kapunan was ordered confined under
“house arrest” by then Chief of Staff (later President) Gen. Fidel Ramos. Kapunan was also ordered, as a condition for his
house arrest, that he may not issue any press statements or give any press conference during his period of detention. The
Court unanimously upheld such restrictions, noting:
“… to a certain degree, individual rights may be curtailed, because the effectiveness of the military in fulfilling its
duties under the law depends to a large extent on the maintenance of discipline within its ranks. Hence, lawful orders
must be followed without question and rules must be faithfully complied with, irrespective of a soldier's personal views on
the matter. It is from this viewpoint that the restrictions imposed on petitioner Kapunan, an officer in the AFP, have to be
considered.”
As a general rule, it is integral to military discipline that the soldier’s speech be with the consent and approval of
the military commander. The necessity of upholding the ability to restrain speech becomes even more imperative if the
soldier desires to speak freely on political matters. For there is no constitutional provision or military indoctrination will
eliminate a soldier’s ability to form a personal political opinion, yet it is vital that such opinions be kept out of the public
eye. For one, political belief is a potential source of discord among people, and a military torn by political strife is
incapable of fulfilling its constitutional function as protectors of the people and of the State. For another, it is ruinous to
military discipline to foment an atmosphere that promotes an active dislike of or dissent against the President, the
commander-in-chief of the armed forces. Soldiers are constitutionally obliged to obey a President they may dislike or
distrust. Even petitioners are well aware that it was necessary for them to obtain permission from their superiors before
they could travel to Manila to attend the Senate Hearing.
Congress holds significant control over the armed forces in matters such as budget appropriations and the
approval of higher-rank promotions, yet it is on the President that the Constitution vests the title as commander-in-chief
and all the prerogatives and functions appertaining to the position. Again, the exigencies of military discipline and the
chain of command mandate that the President’s ability to control the individual members of the armed forces be accorded
the utmost respect. Where a military officer is torn between obeying the President and obeying the Senate, the Court will
without hesitation affirm that the officer has to choose the President. After all, the Constitution prescribes that it is the
President, and not the Senate, who is the commander-in-chief of the armed forces.
Judicial relief as remedy:
The refusal of the President to allow members of the military to appear before Congress is not absolute.
Inasmuch as it is ill-advised for Congress to interfere with the President’s power as commander-in-chief, it is similarly
detrimental for the President to unduly interfere with Congress’s right to conduct legislative inquiries. The impasse did not
come to pass in this petition, since petitioners testified anyway despite the presidential prohibition. The remedy lies with
the courts.
Senate affirmed both the Arnault and Bengzon rulings. It elucidated on the constitutional scope and limitations on
the constitutional power of congressional inquiry. Thus, the power of inquiry, “with process to enforce it,” is grounded on
the necessity of information in the legislative process. If the information possessed by executive officials on the operation
of their offices is necessary for wise legislation on that subject, by parity of reasoning, Congress has the right to that
information and the power to compel the disclosure thereof.
It may thus be subjected to judicial review pursuant to the Court’s certiorari powers under Section 1, Article VIII of
the Constitution. To avoid conflict, Congress must indicate in its invitations to the public officials concerned, or to any
person for that matter, the possible needed statute which prompted the need for the inquiry. Section 21, Article VI likewise
establishes critical safeguards that proscribe the legislative power of inquiry. The provision requires that the inquiry be
done in accordance with the Senate or House’s duly published rules of procedure, necessarily implying the constitutional
infirmity of an inquiry conducted without duly published rules of procedure. Section 21 also mandates that the rights of
persons appearing in or affected by such inquiries be respected, an imposition that obligates Congress to adhere to the
guarantees in the Bill of Rights.
In Senate, the Court ruled that the President could not impose a blanket prohibition barring executive officials
from testifying before Congress without the President’s consent notwithstanding the invocation of executive privilege to
justify such prohibition. Should neither branch yield to the other branch’s assertion, the constitutional recourse is to the
courts, as the final arbiter if the dispute. It is only the courts that can compel, with conclusiveness, attendance or non-
attendance in legislative inquiries.
Courts are empowered, under the constitutional principle of judicial review, to arbitrate disputes between the
legislative and executive branches of government on the proper constitutional parameters of power. By this and, if the
courts so rule, the duty falls on the shoulders of the President, as commander-in-chief, to authorize the appearance of the
military officers before Congress. Even if the President has earlier disagreed with the notion of officers appearing before
the legislature to testify, the Chief Executive is nonetheless obliged to comply with the final orders of the courts.
Lastly, General Gudani argues that he can no longer fall within the jurisdiction of the court-martial, considering
his retirement last 4 October 2005. He cites Article 2, Title I of Commonwealth Act No. 408, which defines persons subject
to military law as, among others, “all officers and soldiers in the active service of the [AFP],” and points out that he is no
longer in the active service. However, an officer whose name was dropped from the roll of officers cannot be considered
to be outside the jurisdiction of military authorities when military justice proceedings were initiated against him before the
termination of his service. Once jurisdiction has been acquired over the officer, it continues until his case is terminated.

31. RANDOLF DAVID, ET AL. VS. GLORIA MACAPAGAL-ARROYO, ET AL. G.R. No. 171396, 171409, 171485,
171483, 171400, 171489 & 171424 May 3, 2006
Presidential Proclamation No. 1017

Facts:
On February 24, 2006, as the nation celebrated the 20th Anniversary of the Edsa People Power I, President
Arroyo issued PP 1017 declaring a state of national emergency and call upon the Armed Forces of the Philippines (AFP)
and the Philippine National Police (PNP), to prevent and suppress acts of terrorism and lawless violence in the country.
The Office of the President announced the cancellation of all programs and activities related to the 20th anniversary
celebration of Edsa People Power I; and revoked the permits to hold rallies issued earlier by the local governments and
dispersal of the rallyists along EDSA. The police arrested (without warrant) petitioner Randolf S. David, a professor at the
University of the Philippines and newspaper columnist. Also arrested was his companion, Ronald Llamas, president of
party-list Akbayan.
In the early morning of February 25, 2006, operatives of the Criminal Investigation and Detection Group (CIDG)
of the PNP, on the basis of PP 1017 and G.O. No. 5, raided the Daily Tribune offices in Manila and attempt to arrest was
made against representatives of ANAKPAWIS, GABRIELA and BAYAN MUNA whom suspected of inciting to sedition and
rebellion. On March 3, 2006, President Arroyo issued PP 1021 declaring that the state of national emergency has ceased
to exist. Petitioners filed seven (7) certiorari with the Supreme Court and three (3) of those petitions impleaded President
Arroyo as respondent questioning the legality of the proclamation, alleging that it encroaches the emergency powers of
Congress and it violates the constitutional guarantees of freedom of the press, of speech and assembly.

Issue:
1.) Whether or not Presidential Proclamation No. 1017 is unconstitutional?
2.) Whether or not the warantless arrest of Randolf S. David and Ronald Llamas and the dispersal of KMU and NAFLU-
KMU members during rallies were valid?
3.) Whether or not proper to implead President Gloria Macapagal Arroyo as respondent in the petitions?
4.) Whether or not the petitioners have a legal standing in questioning the constitutionality of the proclamation?
5.) Whether or not the concurrence of Congress is necessary whenever the alarming powers incident to Martial Law are
used?

Ruling:
1.) The Court finds and so holds that PP 1017 is constitutional insofar as it constitutes a call by the President for the
AFP to prevent or suppress lawless violence whenever becomes necessary as prescribe under Section 18, Article VII of
the Constitution. However, there were extraneous provisions giving the President express or implied power
(A) To issue decrees; (" Legislative power is peculiarly within the province of the Legislature. Section 1, Article VI
categorically states that "[t]he legislative power shall be vested in the Congress of the Philippines which shall consist of a
Senate and a House of Representatives.")
(B) To direct the AFP to enforce obedience to all laws even those not related to lawless violence as well as decrees
promulgated by the President[The absence of a law defining "acts of terrorism" may result in abuse and oppression on the
part of the police or military]; and
(C) To impose standards on media or any form of prior restraint on the press, are ultra vires and unconstitutional. The
Court also rules that under Section 17, Article XII of the Constitution, the President, in the absence of legislative
legislation, cannot take over privately-owned public utility and private business affected with public interest. Therefore, the
PP No. 1017 is only partly unconstitutional.

2.) The warrantless arrest of Randolf S. David and Ronald Llamas; the dispersal and warrantless arrest of the KMU
and NAFLU-KMU members during their rallies are illegal, in the absence of proof that these petitioners were committing
acts constituting lawless violence, invasion or rebellion and violating BP 880; the imposition of standards on media or any
form of prior restraint on the press, as well as the warrantless search of the Tribune offices and whimsical seizure of its
articles for publication and other materials, are declared unconstitutional because there was no clear and present danger
of a substantive evil that the state has a right to prevent.

3.) It is not proper to implead President Arroyo as respondent. Settled is the doctrine that the President, during his
tenure of office or actual incumbency, may not be sued in any civil or criminal case, and there is no need to provide for it
in the Constitution or law.

4.) This Court adopted the “direct injury” test in our jurisdiction. In People v. Vera, it held that the person who impugns
the validity of a statute must have “a personal and substantial interest in the case such that he has sustained, or will
sustain direct injury as a result.” Therefore, the court ruled that the petitioners have a locus standi, for they suffered “direct
injury” resulting from “illegal arrest” and “unlawful search” committed by police operatives pursuant to PP 1017.

5.) Under Article XII Section 17 of the 1987 Philippine Constitution, in times of national emergency, when the public
interest so requires, the President may temporarily take over a privately owned public utility or business affected with
public interest only if there is congressional authority or approval. There must enactment of appropriate legislation
prescribing the terms and conditions under which the President may exercise the powers that will serves as the best
assurance that due process of law would be observed.

32. Jamar M. Kulayan, et al. vs. Gov. Abdusakur M. Tan etc., et al.,
G.R. No. 187298, July 3, 2012.

Executive power; emergency or calling-out powers of President. [I]t has already been established that there is one
repository of executive powers, and that is the President of the Republic. This means that when Section 1, Article VII of
the Constitution speaks of executive power, it is granted to the President and no one else. As emphasized by Justice
Jose P. Laurel, in his ponencia in [Villena v. Secretary of the Interior, 67 Phil. 541 (1939)]: “With reference to the
Executive Department of the government, there is one purpose which is crystal-clear and is readily visible without the
projection of judicial searchlight, and that is the establishment of a single, not plural, Executive. The first section of Article
VII of the Constitution, dealing with the Executive Department, begins with the enunciation of the principle that ‘The
executive power shall be vested in a President of the Philippines.’ This means that the President of the Philippines is the
Executive of the Government of the Philippines, and no other.” Corollarily, it is only the President, as Executive, who is
authorized to exercise emergency powers as provided under Section 23, Article VI, of the Constitution, as well as what
became known as the calling-out powers under Section 7, Article VII thereof.
SECTION 19

33. TORRES VS. GONZALES


(152 SCRA 272, 1987)

FACTS: Petitioner was convicted by the trial court of the crime of estafa and was sentenced to prison term. Petitioner was
later granted a conditional pardon on condition that petitioner would "not again violate any of the penal laws of the
Philippines. Should this condition be violated, he will be proceeded against in the manner prescribed by law." Petitioner
accepted the conditional pardon and was consequently released from confinement.
The Board of Pardons and Parole (the "Board") resolved to recommend to the President the cancellation of the
conditional pardon granted to the petitioner. The evidence before the Board showed that on 22 March 1982 and 24 June
1982, petitioner had been charged with twenty counts of estafa, which cases were then pending trial before the Regional
Trial Court of Rizal. The record also showed that petitioner had been convicted by the trial court of the crime of sedition:
this conviction was then pending appeal before the Intermediate Appellate Court.
Respondent Minister of Justice wrote to the President of the Philippines informing her of the Resolution of the
Board recommending cancellation of the conditional pardon previously granted to petitioner. Thereafter, the President
cancelled the conditional pardon of the petitioner.
Respondent Minister issued "by authority of the President" an Order of Arrest and Recommitment against
petitioner. The petitioner was accordingly arrested and confined in Muntinlupa to serve the unexpired portion of his
sentence.
Petitioner now impugns the validity of the Order of Arrest and Recommitment. He claims that he did not violate
his conditional pardon since he has not been convicted by final judgment of the estafa nor of the crime of sedition.

ISSUE: Whether or not conviction of a crime by final judgment of a court is necessary before the petitioner can be validly
rearrested and recommitted for violation of the terms of his conditional pardon.

HELD: It depends.
The determination of the occurrence of a breach of a condition of a pardon, and the proper consequences of
such breach, may be either a purely executive act, not subject to judicial scrutiny under Section 64 (i) of the Revised
Administrative Code; or it may be a judicial act consisting of trial for and conviction of violation of a conditional pardon
under Article 159 of the Revised Penal Code. Where the President opts to proceed under Section 64 (i) of the Revised
Administrative Code, no judicial pronouncement of guilt of a subsequent crime is necessary, much less conviction therefor
by final judgment of a court, in order that a convict may be recommended for the violation of his conditional pardon.
Here, the President has chosen to proceed against the petitioner under Section 64 (i) of the Revised Administrative Code.
That choice is an exercise of the President's executive prerogative and is not subject to judicial scrutiny.

34. MONSANTO VS. FACTORAN, JR.


(170 SCRA 190, 1989)

FACTS: The Sandiganbayan convicted petitioner Monsanto and three accused of the complex crime of estafa thru
falsification of public documents.
Petitioner Monsanto appealed her conviction to this Court which subsequently affirmed the same. She then filed
a motion for reconsideration but while said motion was pending, she was extended by then President Marcos absolute
pardon which she accepted.
By reason of said pardon, petitioner wrote the Calbayog City Treasurer requesting that she be restored to her
former post as assistant city treasurer since the same was still vacant.
Petitioner‘s letter-request was referred to the Ministry of Finance for resolution. The Finance Ministry ruled that petitioner
may be reinstated to her position without the necessity of a new appointment not earlier than the date she was extended
the absolute pardon. It also directed the city treasurer to see to it that the amount of P4,892.50 which the Sandiganbayan
had required to be indemnified in favor of the government.
Seeking reconsideration of the foregoing ruling, petitioner wrote the Ministry stressing that the full pardon
bestowed on her has wiped put the crime which implies that her service in the government has never been interrupted
and therefore the date of reinstatement should correspond to the date of her preventive suspension; that she is entitled to
backpay for the entire period of her suspension; and that she should not be required to pay the proportionate share of the
amount of P4892.50.
The Ministry of Finance referred petitioner‘s letter to the Office of the President for further review and action.
The respondent Deputy Executive Secretary Factoran denied the petitioner‘s request and holds that Monsanto is
not entitled to an automatic reinstatement on the basis of the absolute pardon granted her, but must secure an
appointment to her former position and that she is liable for the civil liability to her previous conviction.
Petitioner filed a motion for reconsideration which was denied, hence this petition.

ISSUE: W/N a public officer who had been granted an absolute pardon by the Chief Executive is entitled to reinstatement
to her former position without need of a new appointment.

HELD:
NO. Having accepted the pardon, petitioner is deemed to have abandoned her appeal and her conviction by the
Sandiganbayan and has assumed the character of finality.
The essence of pardon is the remission of guilt. Thus pardon implies guilt. Pardon does not ipso facto reinstate a
convicted felon to public office which was forfeited by reason of the conviction. It should be noted that public office is
intended primarily for collective protection, safety and benefit of the common good. Thus, it cannot be compromised to
favor private interests. A pardon does not virtually acquit the accused of the offense charged.
But the pardon restores the petitioner eligibility for appointment to the said office. Thus, to regain her former post,
she must reapply and undergo the usual procedure required for a new appointment.

35. PEOPLE VS. SALLE


(250 SCRA 581, 1995)

FACTS: Accused-appellants Francisco Salle, Jr. and Rickey Mengote were found guilty beyond reasonable, doubt as co-
principals of the compound crime of murder and destructive arson. Appellants seasonably filed their notice of appeal.
However, Salle filed an urgent motion to withdraw his appeal in view of his acceptance of the grant of conditional pardon
by the President. Mengote was also granted a conditional pardon and was released from confinement, but the latter did
not file a motion to withdraw his appeal.
The court granted Salle's motion to withdraw his appeal and his case is considered closed and terminated,
insofar as he is concerned. Mengote, on the other hand, has not filed any motion to withdraw his appeal.

ISSUE: W/N appeal of the case must be withdrawn for the effectivity of a pardon granted.

HELD: YES. The "conviction by final judgment" limitation under Section 19 of Art.VII of the present constitution prohibits
the grant of pardon, whether full or conditional, to an accused during the pending of his appeal from his conviction by the
trial court. Any application therefore, if one is made, should no be acted upon or the process towards its grant should no
begun unless the appeal is withdrawn or the conviction is final.
A judgment of conviction become final (a) when no appeal is seasonably perfected" (b) when the accuse
commences to serve the sentence, (c) when the right to appeal is expressly waived in writing, except where the death
penalty was imposed by the trial court, and (d) then the accused applies for probation, thereby waiving his right to appeal.
Thus, where the judgment of conviction is still pending appeal and has not yet therefore attained finality as in the case of
Mengote, executive clemency may not be granted to appellant
The reason is that, the doctrine of separation of powers of demands that such exclusive authority of the appellate
court be fully respected and kept unimpaired.

36. GARCIA VS. COA


(226 SCRA 356, 1993)

FACTS: Petitioner Vicente Garcia was a supervising lineman of the Bureau of Telecommunication in Lucena City before
he was summarily dismissed on April I, 1975 on the ground of dishonesty in accordance with the decision of the Ministry
of Public Works in Administrative Case No. 975 for the loss of several telegraphs poles. Petitioner did not appeal the
decision.
Based on the same facts, a criminal case for qualified theft was filed against petitioner, which was resolved
acquitting petitioner. In view of his acquittal Garcia sought reinstatement to his former position which was denied by the
Bureau of Telecommunications. Petitioner pleaded to the President of the Philippines for executive clemency which was
granted.
Thereafter, Garcia filed with the respondent COA a claim for payment of back salaries effective April 1975. This
was denied by COA on the ground that the executive clemency granted to him did not provide for the payment of back
salaries and that he had not been reinstated in the service. Petitioner then appealed the COA decision to the Office of the
President. Based on the recommendation of the Deputy Secretary Factoran, petitioner filed this petition for review on
certiorari.

ISSUE: W/N petitioner is entitled to payment of back wages after having been reinstated pursuant to the grant of
executive clemency.

HELD: YES. Petitioner's automatic reinstatement to the government service entitles him to back wages. This is meant to
afford relief to petitioner who is innocent from the start and to make reparation for what he has suffered as a result of his
unjust dismissal from the service. To rule otherwise would defeat the very intention of the executive clemency, i.e., to give
justice to petitioner. Moreover, the right to back wages is afforded to those who have been illegally dismissed and were
thus ordered reinstated or to those otherwise acquitted of the charges against them. There is no doubt that petitioner's
case falls within the situations aforementioned to entitle him to back wages.
The bestowal of executive clemency on petitioner in effect completely obliterated the adverse effects of the
administrative decision which found him guilty of dishonesty and ordered his separation from the service. This can be
inferred from the executive clemency itself exculpating petitioner from the administrative charge and thereby directing his
reinstatement, which is rendered automatic by the grant of the pardon. This signifies that petitioner need no longer apply
to be reinstated to his former employment; he is restored to his office ipso facto upon the issuance of the clemency.

37. LLAMAS VS. ORBOS


(202 SCRA 844, 1991)

FACTS: Petitioner Rodolfo Llamas is the incumbent Vice Governor of the Province of Tarlac. He assumed the position by
virtue of a decision of the office of the President, the governorship. Private respondent Mariano Ocampo III is the
incumbent Governor of the Province of Tarlac and was suspended from office for a period of 90 days due to a verified
complaint filed by petitioner against respondent Governor before the Department of Local Government charging him with
the alleged violation of the Local Government Code and the Anti-graft and Corrupt Practices Law. Public respondent
Oscar Orbos was the Executive Secretary at the time of the filing of this petition and is being impleaded herein in that
official capacity for having issued, by authority of the President, the assailed Resolution granting executive clemency to
respondent governor.
Petitioner's main argument is that the President may grant executive clemency only in criminal cases based on Art. VII,
Sec. 19 of the Constitution. According to the petitioner, the qualifying phrase "after conviction by final judgment applies
solely to criminal cases and no other law allows the grant of executive clemency or pardon to anyone who has been
convicted in an administrative case."

ISSUE: Whether or not the President of the Philippines has the power to grant executive clemency in administrative
cases.

HELD: YES. The President has the power to grant executive clemency in administrative case because the Constitution
does not distinguish between which cases executive clemency may be exercised by the President, with the sole exclusion
of impeachment cases. SC also held that there are no valid and convincing reasons why the President cannot grant
executive clemency in administrative cases, which are clearly less serious than criminal offense.

38. EDUARDO KAPUNAN V COURT OF APPEALS; MARCH 13, 2009


SECTION 21
39. COMMISSIONER OF CUSTOMS VS. EASTERN SEA TRADING
3 SCRA 351[1961]

FACTS: The respondent was the consignee of several shipments of onion and garlic. Since none of the
shipments had the required certificate by the Central Bank Circular Nos. 44 and 45 for the release of goods thus imported
were seized and subjected to forfeiture proceedings. The Collector of Customs of Manila having been in the meantime
released to the consignees on surety bonds directed the same and its surety that the amount of the bonds be paid, jointly
and severally to the Bureau of Customs. The Consignee thereafter sought a review with the Court of Tax Appeal which
reversed the decision and ordered that the bond be withdrawn and cancelled. Hence, the present petition.
The petition is based upon the facts that insofar as the license and a certificate authorizing the importation
release of the goods under the consideration are required by the Central Bank Circulars Nos. 44 and 45, the latter are null
and void and the seizure and the forfeiture of the goods imported from Japan cannot implement an executive agreement-
extending the effectivity of our Trade and Financial agreement with Japan-which is dubious validity, because there is no
government agency authorized to issue the import license required by the aforementioned executive order.

ISSUE: Whether the executive agreement is valid even without the 2/3 concurrence of the Senate.

HELD: YES. The concurrence of the House of Congress is required by our fundamental law in the making of treaties
which are however distinct and different from executive agreements which may validly entered into without such
occurrence.
The validity of the executive agreement in question is thus present. In fact, the so-called parity right provided for
in the ordinance appended to our Constitution were prior thereto, the subject of an executive agreement, made without the
concurrence of2/3s of the Senate of the United States.

Vous aimerez peut-être aussi